Business Finance Chapter 7, Business Finance Chapter 8, Business Finance Chapter 9, Business Finance Final Exam review, Business Finance Exam 3 Final Exam, Finance Test 3, Business Finance Ch. 6, Business Finance Chapter 9, Business Finance 3, Busine...

Lakukan tugas rumah & ujian kamu dengan baik sekarang menggunakan Quizwiz!

Six months ago, you purchased 1,200 shares of ABC stock for $33.56 a share. You have received dividend payments equal to $0.40 a share. Today, you sold all of your shares for $36.70 a share. What is your total dollar return on this investment?

$4,248

Expenses for a 1,000 square foot office space are $6.00/square foot. The lease specifies an expense of $5.40. What is the total expense paid by the landlord?

$5,400 (1000 x 5.40)

If a firm's variable cost per unit estimate used in its base case analysis is $50 per unit and they anticipate the upper and lower bounds to be +/- 10%, What is the "worst case" for variable cost per unit?

$55

You are analyzing a company that has cash of $8,800, accounts receivable of $15,800, fixed assets of $87,600, accounts payable of $40,300, and inventory of $46,900. What is the quick ratio?

0 1.20 O .67 O .83 O .61** 0 1.64 Quick ratio = ($8,800 + 15,800) / $40,300 = .61

Andras Technology has accounts receivable of $35,680, total assets of $538,500, cost of goods sold of $325,400, and a capital intensity ratio of .90. What is the accounts receivable turnover rate?

0 15.56 -..> 0 16.77 0 9.12 0 10.13 0 10.31 Accounts receivable turnover = ($538,500 / .90) / $35,680 = 16.77

The Kids' Mart has a market-to-book ratio of 3.3, net income of $87,100, a book value per share of $18.50, and 7,500 shares of stock outstanding. What is the price-earnings ratio?

0 4.34 0 8.16 0 5.61 0 6.25 0 5.26*** Price-earnings ratio = (3.3 x18.50)/(87100/7500)=5.26

A typical RESPA closing statement has which of the following characteristics?

2 Columns - summary of borrowers and sellers transactions

Which one of the following has the highest effective annual rate? 6 percent compounded annually 6 percent compounded semiannually 6 percent compounded quarterly 6 percent compounded daily 6 percent compounded every 2 years

6 percent compounded daily

What is the annual interest rate of a fully amortizing, 20 year fixed rate 175,000 mortgage, with a monthly payment of $1,266.41? a. 6.434% b. 7.098% c. 6.125% d. 5.098%

6.125%

CGJ Linens has a beta of 1.09. The risk-free rate of return is 2.75 percent and the market rate of return is 9.80 percent. What is the risk premium on this stock?

7.68 percent

The common stock of United Industries has a beta of 1.34 and an expected return of 14.29 percent. The risk-free rate of return is 3.7 percent. What is the expected market risk premium?

7.90 percent

What is the effective annual rate of 8.25 percent compounded quarterly?

8.51 percent

Bud is offering...

9.39%

required return?

= Dividend yield + Capital gains yield

The expected rate of return on a Roth IRA is

A Roth IRA is a tax-shelter, not a type of investment. The expected rate of return depends on what invesment you put in your Roth IRA

Coupon

A bond's _____ payment is a fixed amount of interest that is paid annually or semiannually by the issuer to its bondholders

a fixed amount of interest that is paid annually or semiannually by the issuer to its bondholders

A bonds coupon payment is:

Which is NOT a component of an ARM? (A) A margin (B) An index (C) A chapter (D) Caps

A chapter

Probability Distribution

A representation of possible outcomes (states of nature) that may occur and the likelihood (probability) of each outcome.

Coefficient of Variation(CV)

A standard measure of dispersion about the expected value,that shows the risk per unit of return

makes no interest payments

A zero-coupon bond is a bond that ____.

Single Security and/or Poorly Diversified Portfolio

-Placing entire investment into this, then SD is the appropriate risk measurement.

You can expect greater benefits for using an IRA when you have

-longer time horizons (starting in 20's vs. 40's) -higher tax rates -higher ROR

Capital Budgeting Decision Criteria

-the decision rule should consider all relevant cash flows. -the " " should acknowledge the time value of money concept. -the " " should consider the riskiness of cash flows. -the " " should always rank projects so that those projects that add the most to the value of the firm are ranked highest.

The beta of a portfolio is?

-the sum of the weight of each asset times the beta of each asset. So

The expected return of a portfolio is?

-the sum of the weight of each asset times the expected return of each asset.

The Small Business Administration (SBA) may consider a business with many employees to be small as long as it has low annual revenues.

...

You want to purchase a new condominium that costs $287,500. Your plan is to pay 25 percent down in cash and finance the balance over 15 years at 3.75 percent. What will be your monthly mortgage payment including principal and interest?

A. $1,568.07

What was the rating that was given to many of these securities during the housing boom?

AAA

What organization was founded in 1967 for economic, political, social, and cultural cooperation in the Asia Pacific region?

ASEAN

Which organization was developed to promote economic and cultural cooperation among Asian countries?

ASEAN

An historical summary of the publicly recorded documents that affect the ownership of a property known as a(n):

Abstract

Which one of the following statements best defines the efficient market hypothesis?

All securities in an efficient market are zero net present value investments.

What does a planned economy rely on a centralized government to do?

Allocate all or most factors of production

Probability Distributions

Allow us to calculate the E.R. of an investment and the SD of that investment based on the values of associated with our distribution of possible outcomes.

Diversification

Allows us to greatly reduce our risk by holding a portfolio.

Letitia borrowed $6,000 from her bank two years ago. The loan term is four years. Each year, she must repay the bank $1,500 plus the annual interest. Which type of loan does she have?

Amortized

A few years ago, Simon Powell purchased a home for $195,000. Today, the home is worth $340,000. His remaining mortgage balance is $145,000. Assuming that Simon can borrow up to 75 percent of the market value, what is the maximum amount he can currently borrow against his home?

Amount available for borrowing = (Maximum loan percent × Current market value) - Current loan = (0.75 × $340,000) - $145,000 = $110,000

You are comparing two annuities. Annuity A pays $100 at the end of each month for 10 years. Annuity B pays $100 at the beginning of each month for 10 years. The rate of return on both annuities is 8 percent. Which one of the following statements is correct given this information?

Annuity B has both a higher present value and a higher future value than Annuity A.

Assume you deposit $5,100 at the end of each year into an account paying 9.75 percent interest. How much will you have if you make deposits for 36 years?

Annuity future value $ 1,437,494.47 ±0.01%

Assume you deposit $5,100 at the end of each year into an account paying 9.75 percent interest. How much money will you have in the account in 18 years?

Annuity future value $ 226,848.38 ± 0.1%

You want to invest an amount of money today and receive back twice that amount in the future. You expect to earn 6 percent interest. Approximately how long must you wait for your investment to double in value?

Approximate time period = 72/6 = 12 years

You just received a loan offer from Friendly Loans. The company is offering you $5,000 at 9.3 percent interest. The monthly payment is only $100. If you accept this offer, how long will it take you to pay off the loan?

B. 5.29 years

Which economic system emphasizes the private ownership of most factors of production?

Capitalism

Which country in the Pacific Asia region has one of the world's largest economies?

China

Barnes Enterprises has bonds on the market making annual payments, with 18 years to maturity, a par value of $1,000, and a price of $955. At this price, the bonds yield 9.2 percent. What must the coupon rate be on the bonds?

Coupon rate 8.70 %; Solution w/ diff numbers - Coupon rate=$70.95 / $1,000 Coupon rate = .0710 or 7.10%

working capital management includes which one of the following?

Determining which customers will be granted credit

What is profit?

Difference between revenues and expenses

A certificate of deposit often charges a penalty for withdrawing funds before the maturity date. If the penalty involves two months of interest, what would be the amount for early withdrawal on a $32,000, 8 percent CD?

Early withdrawal penalty = Account value × Annual interest rate × (Number of penalty months / 12) = $32,000 × 0.08 × 2/12 = $426.67

Operating cash flow is a function of:

Earnings Before Interest and Taxes Depreciation Taxes

What term denotes a nation's basis for allocating its resources among its citizens?

Economic system

Income after deducting vacancy that is available to pay expenses is referred to as:

Effective gross income

Assume that the market prices of the securities that trade in a particular market fairly reflect the available information related to those securities. Which one of the following terms best defines that market?

Efficient Capital Market

ownership

Equity represents a(n) _____ interest of a firm.

What are individual beliefs about what is right and wrong or good and bad referred to as?

Ethics

Where is the world's largest free marketplace?

Europe

Globalization refers to the process by which countries around the world are becoming more self-sufficient.

FALSE

In a market economy, there is a centralized government that controls factors of production and decision making.

FALSE

In a market system, individuals are limited as to what they can buy and in how they can spend their money. T/F

FALSE

Major innovations are most likely to come from large corporations.

FALSE

Net Operating Income is the income after deduction of mortgage payments

FALSE

T/F: Residential appraisers use only the sales comparison approach to determine value of the homes they appraise.

FALSE

The capitalization rate is equal to the discount rate minus any expected annual growth in income and property value

FALSE

The technological environment includes only electronics and telecommunications used to perform business activities. T/F

FALSE

A property is purchased for $350,000. Based on an annual growth rate of 3%, the resale value for year 10 would be $456,671

FALSE (1.03^10 x 350,000 = 470,370.73)

What is the general term for resources used by a business to produce a good or service referred to as?

Factors of Proudction

A borrower with an interest-only loan may end up owing more at the end of the loan than the original loan amount True or False

False

A housing bubble occurs when there is a big increase in the supply of homes True or False

False

Because of the flaws with the Payback Period, few firms (less than 25%) use the Payback Period when evaluating capital budgeting decisions

False

Negative amortization reduces the principal value of a loan True or False

False

Residential appraisers use only the sales comparison approach to determine value of the homes they appraise. True and False

False

Which of the following treaties sought to eliminate trade barriers such as tariffs and quotas?

General Agreement on Tariffs and Trade

The average compound return earned per year over a multi-year period is called the __________ average return.

Geometric

What is scenario analysis?

Scenario analysis determines the impact on NPV of a set of events relating to a specific scenario.

Which one of the following is a positively sloped linear function that is created when expected returns are graphed against security betas?

Security Market Line

Which is NOT typically one of the settlement costs that are escrowed over the life of the loan?

Selling commissions

Consider two projects: Project A Project B •PP 2.6 years 2.3 years •IRR 14.5% 17.3% •NPV $15,500 $6,900 Assume that the projects both have a required return of 12% and a critical acceptance level (T) of 2.5 years. If these are independent projects we should

Take both projects A and B Both A and B have a positive NPV (most important) and have IRR greater than the required return. While Project A exceeds the critical acceptance level, Payback Period is too flawed of a model to override the NPV analysis. If there is a conflict between NPV and PP, always side with NPV

When an investor is not interested in owning their own business, why do they need to understand entrepreneurship?

To determine the key characteristics of success

When comparing two individual investments to be held as stand-alone investments (not as part of a well diversified portfolio), standard deviation is a better risk measurement than beta.

True

When dealing with independent projects, the size problem associated with the IRR is no longer relevant.

True

A comparable property has a feature that is superior to the subject property. What adjustment...in the sales comparison approach to value?

Value of the feature would be subtracted from the sales price of the comparable property

Coupon Rate

Which of the following terms apply to a bond?

Market interest rate fluctuations

Which one of the following is the most important source of risk from owning bonds?

-humped -upward sloping -downward sloping

Which three of the following are common shapes for the term structure of interest rates?

Many bond transactions are negotiated privately.

Why is the bond market less transparent than the stock market?

Which one of the following terms applies to a bond that initially sells at a deep discount and only makes one payment to bondholders?

Zero coupon

Investment banker

a financial specialist who underwrites and distributes new securities and advises corporate clients about raising new funds.

Negotiated purchase

a firm that needs funds makes contact with an investment banker, and deliberations concerning the new issue begins

Liquidity

a firm's ability to pay its bills on time

Debt ratio

a firm's total liabilities divided by its total assets

Return on equity

a firms net income divided by its common book equity, also called stockholders equity

Syndicate

a group of investment bankers who contractually assist in the buying and selling of a new security issues

Mortgage

a loan to finance real estate in which the lender has first claim on the property

bond rating

are designed to reflect the probability of a bond issue going into default.

which one of the following is most apt to align management's priorities with shareholders' interest?

compensating managers with share stock must be held for a minimum of three years

Current assets

consist primarily of cash, marketable securities, accounts receivable, inventories, and prepaid expenses

What are products that are created abroad and then transported and sold domestically?

Imports

What factor of production is used to generate forecasts based on specialized knowledge and economic data?

Information resources

The future value is directly related to?

Interest Rate

A variable operating cost for a vehicle is

Maintenance and repairs.

Which of the following is not a tax deductible for homeowners? (A) Points in mortgage loans (B) Mortgage interest (C) Property taxes (D) Maintenance expenses

Maintenance expenses

An appraisal usually contains three approaches to valuation. What are those three valuations?

Market Approach The Cost Approach The Income Approach

with a lag to some base rate

Most of the time, a floating-rate bond's coupon adjusts ____.

Choose the correct statement regarding expected return

None of the other answers is correct

Which one of the following is defined by its mean and its standard deviation?

Normal Distribution

Which one of the following statements is correct?

The greater the volatility of returns, the greater the risk premium.

(1 + R) = (1 + r)×(1 + h)

The model that precisely specifies the relationship between the nominal rate and the real rate is: R = the nominal rate r = the real rate h = the rate of inflation

inflation

The nominal rate is found by adding the _____ and the real rate of return.

Smiling Elephant, Inc., has an issue of preferred stock outstanding that pays a $5.10 dividend every year, in perpetuity. If this issue currently sells for $80.15 per share, what is the required return?

The price of a share of preferred stock is the dividend divided by the required return. This is the same equation as the constant growth model, with a dividend growth rate of zero percent. Remember, most preferred stock pays a fixed dividend, so the growth rate is zero. This is a special case of the dividend growth model where the growth rate is zero, or the level perpetuity equation. Using this equation, we find the price per share of the preferred stock is: R = D / P0 R = $5.10 / $80.15 R = .0636, or 6.36%

relationship between short-term and long-term interest rates

The term structure of interest rates examines the ____.

What marks the key difference between economic systems?

The way the factors of production are managed

Pier Imports has cash of $41,100 and accounts receivable of $54,200, all of which is expected to be collected. The inventory cost $82,300 and can be sold today for $116,500. The fixed assets were purchased at a total cost of $234,500 of which $118,900 has been depreciated. The fixed assets can be sold today for $138,000. What is the total book value of the firm's assets?

O $327,800 $293,200** O $346,800 O $412,100 O $415,600 Total book value = $41,100 + 54,200 + 82,300 + 234,500 - 118,900 = $293,200

Northern Lighting purchased some three-year MACRS property three years ago. What is the current book value ofthis equipment if the original cost was $385,000? The MACRS allowance percentages are as follows, commencing with Year 1: 33.33, 44.45, 14.81, and 7.41 percent.

O $57,037.75 O $28,528.50 O $85,547.00 O $96,250.00 Book value3 $385, 000 x (1 . 3333- . 1481) $28, 528 . 50

Capital Budgeting Decision Techniques

Payback Period; Net Present Value; Internal Rate of Return

Under which scenario is negative amortization likely to occur? Payment Cap / Interest Rates (A) None / Increasing (B) None / Decreasing (C) 7.5% / Increasing (D) 7.5% / Decreasing

Payment Cap: 7.5% Interest rates: Increasing

Carrie bought a house five years ago for $150,000. At that time she borrowed $140,000 from her bank. The house is now worth $162,000. Her PMI will automatically be dropped when her mortgage balance drops to

Per the Homeowners Protection Act, PMI must be terminated automatically when a home owner's equity reaches 22% of the property value at the time the mortgage was executed. The mortgage would then be $150,000 × (1 - .22) = $117,000.

You expect to receive $36,000 at graduation in two years. You plan on investing it at 9.25 percent until you have $171,000. How long will you wait from now?

Period 19.61 years

Which aspect of the management process involves determining what the organization needs to do and how best to get it done?

Planning

Income statement

indicates the amount of profits generated by a firm over a given time period

investment risk

is related to the probability of earning a low or negative actual return, the greater the chance of lower than expected, or negative return, the riskier the investment.

When Imogene brought a necklace to a _______, she received a loan based on its value

pawnshop

income bond

pay interest only when interest is earned by the firm

diversifiable risk

portion of a security's stand alone risk that can not be eliminated through proper diversification

Bond Market

primarily traded in over the counter market , most bonds are owned by and traded among large financial institutions. The wall street reports key developments in treasury, corporate , municipal markets.

Though depreciation is a non-cash expense, it is important to capital budgeting for these reasons:

it determines the book value of assets which affects net salvage value; it affects a firm's annual tax liability; it determines taxes owed on fixed assets when they are sold

Economic value added

measures a company's economic profits, rather than allowing profit, in a given year

standard deviations

measures total, or stand-alone risk, the larger the standard deviation is the lower the probabity that actaul returns will be close to expected return, the larger associated with a wider probability distribution of returns

other factors affecting default risk

miscellaneous qualitative factors -earning stability -regulatory environment -potential antitrust or product liabilites -pension liabilities -potential labor problems

Accounts receivable

money owed by customers who purchased goods or services from the firm on credit

Which of the following techniques will provide the most consistently correct result?

net present value (NPV)

Accounts receivable and accounts payable are included in project cash flow estimation as part of changes in ____________

net working capital

S

corporation-a corporation that, because of specific qualifications, is taxed as though it were a partnership

Which of the following are fixed costs?

cost of equipment; rent on a production facility

Fixed costs

costs that do not change as the quantity of output produced changes

An unexpected decrease in market interest rates will cause a:

coupon bond's yield to maturity to decrease.

Investment in net working capital arises when ___.

credit sales are made inventory is purchased cash is kept for unexpected expenditures

Retained earnings

cumulative profits retained in a business up to the date of the balance sheet

What is net working capital?

current assets minus current liabilities

The current yield on a bond is equal to the annual interest divided by the:

current market price.

Earnings before taxes

operating income minus interest expense

The most valuable alternative that is given up if an investment is undertaken is called what?

opportunity cost

A call provision grants the bond issuer the:

option of repurchasing the bonds prior to maturity at a prespecified price.

Miller Farm Products is issuing a 15-year, unsecured bond. Based on this information, you know that this debt can be described as a:

debenture.

sources of capital

debt,preffered stock, common equity, notes payable, longterm debt, retained earining, new common stock.

All else held constant, the present value of an annuity will decrease if you:

decrease the annuity payment.

What is meant by "highest and best use"?

determining highest total land and property value by possible use

Computing the present value of a future cash flow to determine what that cash flow is worth today is called:

discounted cash flow valuation.

Lucas expects to receive a sales bonus of $7,500 one year from now. The process of determining how much that bonus is worth today is called:

discounting.

What is the point at which the supply curve and the demand curve intersect on a graph?

equilibrium

the Sarbanes- Oxley Act of 2002 has:

essentially made officers of publicly traded firms personally responsible for the firm's financial statements.

Accrued expenses

expenses that have been incurred but not yet paid in cash

True or false: Fixed costs cannot be changed over the life of the investment.

false

True or false: In calculating cash flows, you should consider all financing costs.

false

True or false: Operating cash flow is based on the salvage value of equipment.

false

True or false: The depreciation tax shield is the depreciation deduction divided by the tax rate.

false

` True or false: The value of managerial options is taken into account when performing conventional NPV analysis.

false

Capital gain

gains from selling any asset that is not part of the ordinary

The average compound return earned per year over a multi-year period is called the _____ average return.

geometric

The yield to maturity on a discount bond is:

greater than both the current yield and the coupon rate.

Taxable income

gross income from all sources, except for allowable exclusions, less any tax-deductible expenses

Preferred stockholders

have claims on the firm's income and assets after creditors but before common stockholders

If we find that our estimated NPV is sensitive to a variable that is difficult to forecast, then the degree of forecasting risk is

high

What will not result in an increase in housing demand?

higher interest rates

Assume all else is equal. When comparing savings accounts, you should select the account that has the:

highest effective annual rate.

Operations management

how effectively management is performing in day-to-day operations of generating revenues and controlling costs and expenses

preferred stock

hybrid security, like bonds, preferred stockholders recieve a fixed dividend that must be paid before dividends are paid to common stockholders, companies can omit preferred dividened payments without fear of pushing the firm into bankruptcy .

in equilibrium we assume that a stock's price equals its intrinsic value

outsiders estimate intrinsic value to help determine which stocks are attractive to buy and or sell, stocks with a price below (above) its intrinsic value are undervalued(overvalued)

Default Risk

if an issuer defaults, investors recieve less than the promised return. Therefore , the expected return on corporate and municipal bonds is less than the promised return. Influenced by the issuer's financial strength and the terms of the bond contract.

When a homeowners improves some aspect of his property far in excess of competition...

over-improved the property

Comments on Beta

if beta=1.0 the security is just as risky as the average stock. if beta >1.0 the security is riskier than average, if beta < 1.0 the security is less risky than average. Most stocks have betas in the range of 0.5 to 1.5

Interest expenses incurred on debt financing are ______ when computing cash flows from a project.

ignored

Callable bonds

include a deferred call provision and declining call premium.

Which of the following is an example of a sunk cost?

project consultation fee

The information on the specifics of a particular mutual fund (costs, risks, objective, past performance, etc.) can be found in the

prospectus

The primary purpose of bond covenants is to:

protect the bondholders.

Scott borrowed $2,500 today at an APR of 7.4 percent. The loan agreement requires him to repay $2,685 in one lump sum payment one year from now. This type of loan is referred to as a(n):

pure discount loan.

An option on a real asset rather than a financial asset is known as a:

real option; managerial option

Erosion will ______ the sales of existing products.

reduce

The supply of space is

relatively inelastic in the short run, and highly elastic in the long run

Opportunity costs are classified as ____ costs in project analysis.

relevant

The first step in estimating cash flow is to determine the _________ cash flows.

relevant

Gross profit

sales (revenue) minus the cost of goods sold

Operating income

sales less the cost of goods less operating expenses

The influence on property values brought about by a new benefit related to the value of public goods less their cost is referred to as:

the capitalization effect

Net working capital

the difference between a firm's current assets and its current liabilities

risk premium

the difference between the return on a risky asset and a riskless asset, which serves as compensation for investors to hold riskier securities.

Nominal rate of interest

the interest paid on debt securities w/o an adjustment for any loss in purchasing power

Opportunity cost of funds

the next-best rate of return available to the investor for a given

Accumulated depreciation

the sum of all depreciation expense taken for a depreciable asset up to the date of the financial statement.

Flotation costs

the transaction costs incurred when a firm raises funds by issuing a particular type of security

Failure to diversify part 2

there can only be one price(the market return) for a given security, no compensation should be earned for holding unnecessary , diversifiable risk.

Standard deviation measures which type of risk?

total

Estimates of which of the following are needed to prepare pro forma income statements?

unit sales; variable costs; selling price per unit

Which one of the following risks is irrelevant to a well-diversified investor?

unsystematic risk

The basic approach to evaluating cash flow and NPV estimates involves asking:

what-if questions

Theo's BBQ has $48,000 in current assets and $39,000 in current liabilities. Decisions related to these accounts as referred to as:

working captial management

The market-required rate of return on a bond that is held for its entire life is called the:

yield to maturity

As a result of being an armed services veteran, Dan should be eligible for a(n)

VA loan

A borrower has secured a 30 year, $150,000 loan at 7% with monthly payments. Fifteen years later an investor wants to purchase the lender. If market rates are 5%, what would the investor be willing to pay for the loan? A:$75,000 B:$111,028 C:$126,196.48 D:$168,646

$126,196.48

A loan was made 10 years ago for $140,000 at 10.5% for a 30 year term. Rates are currently ... what is the market value of the loan?

$139,828

An ATM with a service fee of $3 is used by a person 100 times in a year. What would be the future value in 7 years (use a 2 percent rate) of the annual amount paid in ATM fees?

2,230.20 Annual fee = Service fee per transaction × Number of transactions per year = $3 × 100 = $300 Future value = Annual fee × FV factor = $300 × 7.434 = $2,230.20

Why do attitudes about entrepreneurship vary internationally?

Attitudes towards risk-taking in business are culturally determined.

How do top managers BEST demonstrate a commitment to ethical business practices?

By adopting and enforcing written codes of ethics

Which one of the following is the quoted price of a bond?

Clean price

CDO stands for:

Collateralized Debt Obligation

Comparing actual performance against standards is an example of which function of the management process?

Controlling

What function of the management process is used to monitor and ensure the organization is meeting its goals?

Controlling

Which one of the following is the price that an investor pays to purchase an outstanding bond?

Dirty price

The discount rate is a rate that a typical investor would normally require as a(n) _____ investment holding period

Expected

Shareholders probably have the most interest in which one of the following sets of ratios?

Return on Equity and Price-Earnings

Market risk impacts all stocks but not equally

True

Opportunity costs are ____.

benefits lost due to taking on a particular project

The owner/landlord is known as the

Lessor

How well does the IRR meet criteria?

Very well if projects are independent. If they are M.E. not so well. If projects are independent (& there is no crossover problem) the IRR will always make the right decision.

PK Software has 7.8 percent coupon bonds on the market with 25 years to maturity. The bonds make semiannual payments and currently sell for 108.75 percent of par. What is the YTM?

Yield to maturity 7.06 %

SD Equation

o= SQUARE ROOT OVER ALL! P1(k1 - k)squared + P2(k2 - k)squared + ....... Pn(kn - k)squared -Sigma (o)= SD -Pi= the probability of the ith outcome -ki= the return under of the ith outcome -k= the expected return for the stock -Pn= the probability of the nth outcome -kn= the return under the nth outcome

By definition, a bank that pays simple interest on a savings account will pay interest:

only on the principal amount originally invested.

Unsystematic risk:

can be effectively eliminated by portfolio diversification.

Operating profit margin

operating income divided by sales

Which of the following are responsible for production and quality control?

operations managers

Side effects from investing in a project refer to cash flows from:

erosion effects; beneficial spillover effects

Price analysis is part of the ____ phase of the research-based buying process.

evaluating alternatives

Best efforts basis

investment banker attempts to sell the issue in return for a fixed commission on each security sold

When Sam applied for a loan, he was assured that his rate would not change if he closed within 30 to 90 days. Sam had a(n) ______ on the interest rate.

lock

Money market

market for short term (less than 1 year) debt instruments

When calculating the cash equivalent of an assumable loan, you find the present value of payments using the

market interest rate

A positive NPV exists when the market value of a project exceeds its cost. Which of these two values is the most difficult to establish?

market value

Beta

measures a stock's market risk,and shows a stock's volatility relative to the market, indicates how risky a stock is if the stock is held in a well-diversified portfolio

general comments

most stocks are positively (through not perfectly) correlated with the market, combining stocks in a portfolio generally lowers risk

High Tech

moves with economy and has a positive correlation. This is typical

Last year, T-bills returned 2 percent while your investment in large-company stocks earned an average of 5 percent. Which one of the following terms refers to the difference between these two rates of return?

risk premium

Bankrupty (liquidation)

unsecured creditors generally receive nothing. this makes them more willing to participate in reorganization even though their claims are greatly scaled back.

In order to analyze the risk of a project's NPV estimate, we should establish ___________ for each important estimate variable.

upper and lower bounds

The different between the existing stock of space and the equilibrium occupancy is know as

vacancy

Standard deviation is a measure of which one of the following?

volatility

You are considering buying a foreclosed home as an investment and expect to hold it for one year before selling it. You require a 25% rate of return on your investment. You are borrowing $110,000 as a interest only loan to make the purchase. Your acquisition costs will be $18,500, your holding costs will be $73,000 and your expected sales costs will be $7,500. What must you sell the home for to earn 25%? A. 200,000 B. 223,167 c. 250,546 d. 214,095

$223,167

The Egg House just borrowed $660,000 to build a new restaurant. The loan terms call for equal annual payments at the end of each year. The loan is for 15 years at an APR of 8.35 percent. How much of the first annual payment will be used to reduce the principal balance?

$23,653.18

Jeffries & Sons is borrowing $95,000 for four years at an APR of 7.05 percent. The principal is to be repaid in equal annual payments over the life of the loan with interest paid annually. Payments will be made at the end of each year. What is the total payment due for Year 3 of this loan?

$27,098.75

You want to have $35,000 in cash to buy a car 3 years from today. You expect to earn 3.6 percent, compounded annually, on your savings. How much do you need to deposit today if this is the only money you save for this purpose?

$31,476.67

Leslie Printing has net income of $26,310 for the year. At the beginning of the year, the firm had common stock of $55,000, paid-in surplus of $11,200, and retained earnings of $48,420. At the end of the year, the firm had total equity of $142,430. The firm paid dividends of $32,500. What is the amount of the net new equity raised during the year?

$34,000** O $42,500 O $25,000 O $21,500 Net new equity = $142,430 - 55,000 - 11,200 - ($48,420 + 26,310 - 32,500) = $34,000

You want your portfolio beta to be 0.90. Currently, your portfolio consists of $4,000 invested in stock A with a beta of 1.47 and $3,000 in stock B with a beta of 0.54. You have another $9,000 to invest and want to divide it between an asset with a beta of 1.74 and a risk-free asset. How much should you invest in the risk-free asset?

$5,034.48

Kevin just deposited $13,000 into his savings account at Traditions Bank. The bank will pay .87 percent interest, compounded annually, on this account. How much interest on interest will he earn over the next 5 years?

$9.93

The subject property of an appraisal has only two bedrooms, but one of the comparables used in the appraisal has three. If the adjustment for a third bedroom is $5,000, the adjustment would be: (A) A $5,000 increase to the comparable's selling price (B) A $5,000 decrease to the comparable's selling price (C) A $5,000 increase to the subject's selling price (D) A $5,000 decrease to the subject's selling price.

(B) $5,000 decrease to the comparables selling price

A house is for sale for $250,000. You have a choice of two 20 year mortgage loans with monthly payments (1) if you make a down of $25,000 you can obtain a loan with a 6% rate of interest or (2) if you make a down payment of $50,000, you can obtain a loan with 5% rate of interest. What is the effective annual rate of interest on the additional $25,000 borrowed on the first loan? (A) 1.00% (B) 6.00% (C) 12.95% (D) 18.67%

(B) 12.95%

If one of the terms of an ARM read, interest is capped at 2%/5%, what would that mean? (A) The borrower can choose the cap he wants by simply circling the appropriate choice (B) The interest rate has a 2% annual cap rate and a 5% lifetime cap rate (C) The interest rate has a 5% annual cap rate and a 2% lifetime cap rate (D) The interest rate has a 2% annual cap rate and a 5% floor cap rate

(B) The interest rate has a 2% annual cap rate and a 5% lifetime cap rate

discount rate

(ri) is the opportunity cost of capital , and is the rate that could be earned on alternative investments of equal risk.

Consider two projects: Project A Project B •PP 2.6 years 2.3 years •IRR 14.5% 17.3% •NPV $15,500 $6,900 Assume that the projects both have a required return of 12% and a critical acceptance level (T) of 2.5 years. If these are mutually exclusive projects we should

Accept Project A and reject Project B Mutually exclusive implies we can do one or the other, but not both (although we could reject both). Since we can only take one, we need to choose the best one. NPV is our best, most reliable decision tool so we want to rely on NPV when dealing with Mutually Exclusive projects. Despite Project B having a better IRR and PP, Project A has a higher NPV and will do more to help us maximize shareholder wealth...so choose A.

You have just made your first $4,000 contribution to your individual retirement account. Assume you earn an annual return 11.35 percent and make no additional contributions. What will your account be worth when you retire in 45 years?

Account value $ 504,800.70

Sweet Treats pays a constant annual dividend of $2.38 a share and currently sells for $52.60 a share. What is the rate of return?

0 4.56 percent 5.39 percent 0 4.52 percent 0 4.83 percent 0 5.91 percent $2.38 / $52.60 = .0452, or 4.52 percent

If sales are $211,000, the profit margin is 6.3 percent, and the capital intensity ratio is .94, what is the return on assets?

0 442 percent 6.08 percent 6.39 percent 6.92 percent 0 6.70 percent** Return on assets = (.063 x x $211,000) - .0670, or 6.70 percent

Fried Donuts has sales of $764,900, total assets of $687,300, total equity of $401,300, net income of $68,200, and dividends paid of $27,000. What is the internal growth

0 5.48 percent 0 6.38 percent*** 0 5.98 percent 0 7.34 percent 0 7.92 percent Internal growth rate = x [($68,200 - x [($68,200 - = .0638, or 6.38 percent

A firm has a return on equity of 17.8 percent, a return on assets of 11.3 percent, and a 65 percent dividend payout ratio. What is the sustainable growth rate?

0 5.72 percent 0 6.84 percent 0 7.12 percent 11.38 percent 0 6.64 percent**** Sustainable growth rate = [.178 x (1 - - [.178 x (1 - .65)]) — - .0664 or 6.64 percent

Galaxy Sales has sales of $938,300, cost of goods sold of $764,500, and inventory of $123,600. How long on average does it take the firm to sell its inventory?

0 6.40 days 0 7.23 days 0 48.68 days 59.01 days**** 0 61.10 days Days' sales in inventory = 365 / ($764,500 / $123,600) = 59.01 days

Rogers Radiators has net income of $48,200, sales of $947,100, a capital intensity ratio of .87, and an equity multiplier of 1.53. What is the return on equity?

0 6.77 percent 0 5.93 percent 0 8.95 percent ** 12.21 percent 0 14.09 percent Return on equity = x (1/.87) x 1.53 - .0895, or 8.95 percent

Calculate the costs of buying versus leasing a motor vehicle.

Buying: Purchase cost = Down payment + [Down payment × Interest rate × (Number of months / 12)] + (Loan payment × Number of payments) - Ending vehicle value = $3,900 + [$3,900 × 0.04 × (48 / 12)] + ($1,170 × 48) - $4,800 = $55,884 Leasing: Lease cost = [Security deposit × Interest rate × (Number of months / 12)] + (Lease payment × Number of payments) + End-of-lease charges = [$1,300 × 0.04 × (48 / 12)] + ($1,170 × 48) + $720 = $57,088

a. What is the total cash outflow for buying and for leasing a motor vehicle with a cash price of $27,200?

Buying: Total cash outflow = Down payment + (Loan payment × Number of payments) - Ending vehicle value = $4,600 + ($640 × 48) − $8,400 = $26,920 Leasing: Total cash outflow = Down payment + (Lease payment × Number of payments) + End-of-lease charges = $1,500 + ($500 × 48) + $750 = $26,250

Tips for Mutual Fund Investing

1. Consider your goals first 2. Bad performance persist 3. Dont chase hot funds 4. Focus on cost 5. Think long-term 6. Consider some International Expense

If you leave the company prior to retirement, you have several options:

1. Leave your $ with the company. 2. Move your $ to your new employer's 401k plan 3. Use a Rollover IRA 4. Cash Out- significant tax penalties.

Major Mutual Fund Categories

1. Money Market Mutual Funds- extremely low risk investments, but also offer low rates of return. Extremely rare to lose money. Yielding < 1% of annual ROR. 2. Bond Funds- Vary in risk. Low would be treasury high would be junk bonds. Can lose money when interest rates increase and/or the bonds suffer from defaults. Average returns of 4-9%. 3. Stock Funds- Most common type. Riskiest type of fund bc its investing in common stocks. Earns rate of 7-13%. 4. Hybrid Funds- requires little management. But, more expensive, less control, cant pick specific stock based funds

A 12-year, annual coupon bond is priced at $1,102.60. The bond has a $1,000 face value and a yield to maturity of 5.33 percent. What is the coupon rate?

6.51 percent; PMT = $65.09 Coupon rate = $65.09 / $1,000 = .0651, or 6.51 percent

You would like to combine a risky stock with a beta of 1.68 with U.S. Treasury bills in such a way that the risk level of the portfolio is equivalent to the risk level of the overall market. What percentage of the portfolio should be invested in the risky stock?

60 percent

The 8 percent, $1,000 face value bonds of Sweet Sue Foods are currently selling at $1,057. These bonds have 16 years left until maturity. What is the current yield?

7.57 percent; Current yield = (.08 ×$1,000)/$1,057 Current yield = .0757 , or 7.57 percent

A stock had returns of 8 percent, -7 percent, 3 percent, and 11 percent over the past 4 years. What is the standard deviation of this stock for the past four years?

7.9%

A firm has inventory of $46,500, accounts payable of $17,400, cash of $1,250, net fixed assets of $318,650, long-term debt of $109,500, and accounts receivable of $16,600. What is the common-size percentage of the equity?

70.60 percent 0 70.12 percent 0 66.87 percent**** 42.08 percent 0 68.75 percent Total assets = Total liabilities and equity = $1,250 + 16,600 + 46,500 + 318,650 = $383,000 Equity common-size percentage = ($383,000 - 17,400 - 109,500) / $383,000 = .6687, or 66.87 percent

Following the signing of the Bankruptcy Abuse Prevention and Consumer Protection Act of 2005, debtors seeking to erase all debts will have to wait ___ year(s) from their last bankruptcy before they can file again.

8 years

A borrower made a mortgage loan 7 years ago for $160,000 at 10.25% interest for 30 years. The loan balance is now $151,806.62 and rates for this amount are currently 9.0% for 23 years. Origination fees and closing costs are $4,500 and closing costs are not financed by the lender. What is the effective cost of refinancing? (A) 9.00% (B) 10.85% (C) 15.32% (D) 9.39%

9.39%

Bud is offering a house for sale for $180,000 with an assumable loan which was made 5 years ago for $140,000 at 8.75% over 30 years. Kelsey is interested in buying the property and can make a $20,000 down payment. A second mortgage can be obtained for the balance at 12.5% for 25 years. What is the effective cost of the combined loans, if Kelsey would like to compare this financing alternative to obtaining a first mortgage for the full amount? (A) 9.00% (B) 10.85% (C) 15.32% (D) 9.39%

9.39%

capital gains yeild?

=Percentage increase in stock price, is the same as the dividend growth rate.

To calculate the dollar return?

=we multiply the number of shares owned by the change in price per share and the dividend per share received.

Exchange Traded Fund (EFT's)

A stock/mutual fund hybrid. It trades like a stock in that you can buy/sell it at any time of the trading day (mutual fund contributions and withdraws are only done based on the end-of-the-day pricing), but each share you buy represents a portfolio of underlying stocks. Tend to have relatively low expense ratios and tend to use more passive stock selection.

Kurt wants to have $835,000 in an investment account six years from now. The account will pay .67 percent interest per month. If he saves money every month, starting one month from now, how much will he have to save each month to reach his goal?

A. $9,062.07

In March 2015, Daniela Motor Financing (DMF), offered some securities for sale to the public. Under the terms of the deal, DMF promised to repay the owner of one of these securities $5,000 in March 2050, but investors would receive nothing until then. Investors paid DMF $1,500 for each of these securities; so they gave up $1,500 in March 2015, for the promise of a $5,000 payment 35 years later. c. In 2022, instead of cashing in the bond for its then current value, you decide to hold the bond until it matures in 2050. What annual rate of return will you earn over the last 28 years?

Rate of return 4.08 %

What is the Market?

Refers to a portfolio of all investment assets.

Diversifiable Risk

Refers to risk factors that are isolated towards one particular firm or industry.

Vesting

Refers to the concept that your retirement benefits belong to you even if you leave the firm before retirement. Minimum vesting standards that require you to be fully vested in 3-6 years. If you are 60% vested, then 60% of your retirement benefits belong to you if you leave the firm early. It is only the matching contributions and the return on those contributions that need to be vested.

Which of the following is an example of an opportunity cost?

Rental income likely to be lost by using a vacant building for an upcoming project

A five-year project is expected to generate annual revenues of $159,000, variable costs of $72,500, and fixed costs of $15,000. The annual depreciation is $19,500 and the tax rate is 21 percent. What is the annual operating cash flow? O $71,500 O $117,855 O $72,430 O $41,080 $60,580 OCF $ 60,580.00 4.

Revenue 159000.00 Variable Costs- 72,500.00 Fixed Costs- 15,000.00 Depreciation- 19,500.00 ___________________________________ EBT$ 52,000.00 ---------------------------- Taxes (21%)- 10,920.00 ---------------------------- Net Income$ 41,080.00 -------------------------- Depreciation19,500.00

Hot Wings, Inc., has an odd dividend policy. The company has just paid a dividend of $8 per share and has announced that it will increase the dividend by $6 per share for each of the next four years, and then never pay another dividend. If you require a 14 percent return on the company's stock, how much will you pay for a share today? Explanation: The price of a stock is the PV of the future dividends. This stock is paying four dividends, so the price of the stock is the PV of these dividends discounted at the required return.

So, the price of the stock is: P0 = $14 / 1.14 + $20 / 1.142 + $26 / 1.143 + $32 / 1.144 P0 = $64.17

The principle of diversification tells us that:

Spreading an investment across many diverse assets will eliminate some of the total risk

Wesen Corp. will pay a dividend of $3.40 next year. The company has stated that it will maintain a constant growth rate of 4.5 percent a year forever. If you want a return of 15 percent, how much will you pay for the stock? If you want a return of 9 percent, how much will you pay for the stock?

Here, we need to value a stock with two different required returns. Using the constant growth model and a required return of 15 percent, the stock price today is: P0 = D1 / (R - g) P0 = $3.40 / (.15 - .045) P0 = $32.38 And the stock price today with a required return of 9 percent will be: P0 = D1 / (R - g) P0 = $3.40 / (.09 - .045) P0 = $75.56

Wesen Corp. will pay a dividend of $3.70 next year. The company has stated that it will maintain a constant growth rate of 5.25 percent a year forever. If you want a return of 18 percent, how much will you pay for the stock? If you want a return of 11 percent, how much will you pay for the stock?

Here, we need to value a stock with two different required returns. Using the constant growth model and a required return of 18 percent, the stock price today is: P0 = D1 / (R - g) P0 = $3.70 / (.18 - .0525) P0 = $29.02 And the stock price today with a required return of 11 percent will be: P0 = D1 / (R - g) P0 = $3.70 / (.11 - .0525) P0 = $64.35

Sarbanes Oxley Act

Holds corporate advisers who have access to or influence on company decisions and boards of directors legally accountable for any instances of misconduct

If you miss payments on a home equity loan, you can lose your

House.

What is the term for the physical and intellectual contributions of people while engaged in the production of goods and services?

Human resources

Which of the following statements related to market efficiency tend to be supported by current evidence? I. Markets tend to respond quickly to new information. II. It is difficult for investors to earn abnormal returns. III. Short-run prices are difficult to predict accurately based on public information. IV. Markets are most likely weak form efficient.

I, II, and III only

The expected return on a portfolio considers which of the following factors? I. percentage of the portfolio invested in each individual security II. projected states of the economy III. the performance of each security given various economic states IV. probability of occurrence for each state of the economy

I,II,III,andIV

Once cash flows have been estimated, which of the following investment criteria can be applied to them?

IRR; NPV; payback period

less than

If a $1,000 par value bond is trading at a discount, it means that the market value of the bond is ______$1,000.

0%

If you are holding a municipal bond that is trading at par to yield 6%, by how much will your after-tax yield change if your federal income tax bracket increases from 15% to 20%. Assume there are no state or local taxes

The bond with a 5% coupon rate

If you are holding two bonds - one with a 5% coupon rate and the other with an 8% coupon rate - which one is more sensitive to interest rate risk, all other things being equal?

The 10 year bond

If you are holding two identical bonds, except that one matures in 10 years and the other matures in 5 years, which bond's price will be more sensitive to interest rate risk?

5%

If you are in the 20% federal income tax bracket, what is your after-tax yield on a municipal bond that is currently trading at par to yield 5%. Assume there are no state or local taxes.

Choose the correct statement regarding vesting

If you are vested, all of your contributions, the company's matching contributions and the returns from those contributions belong to you when you leave the firm.

Standard Deviation

Measures the variability of possible returns and it represented by the sigma. The smaller the SD, the more likely we are going to earn something "close" to our expected return. The greater the SD, the greater the chance that we may earn something far more (good) or far less (bad) than our expected return.

SD

Measures total risk (diversifiable risk + market risk) for a security.

Which type of manager is responsible for implementing the strategies, policies, and decisions made by top managers?

Middle manager

Titles such as plant manager, operations manager, and division manager designate which level of management?

Middle managers

What type of economy is the United States reflective of?

Mixed

Based on Exhibit 7-7, what would be the monthly mortgage payments for each of the following situations?

Monthly mortgage payment = Mortgage payment factor × (Mortgage amount / $1,000) Monthly mortgage payment = $7.91 × ($140,500 / $1,000) = $1,111.36 Monthly mortgage payment = $5.68 × ($215,500 / $1,000) = $1,224.04 Monthly mortgage payment = $6.60 × ($166,000 / $1,000) = $1,095.60

You want to buy a new sports coupe for $74,500, and the finance office at the dealership has quoted you a loan with an APR of 6.9 percent for 36 months to buy the car. What will your monthly payments be?

Monthly payment $ 2,296.94 ± 0.1%

If an adjustable-rate 20-year mortgage for $128,500 starts at 5.0 percent and increases to 5.5 percent, what is the increase in the monthly payment amount?

Monthly payment increase = (New mortgage factor - Old mortgage factor) × (Mortgage amount / $1,000) = ($6.88 − $6.60) × ($128,500.00 / $1,000) = $35.98

What is the main objective of FTL legislation?

More informative disclosure of credit

Which of the following is the main objective FTL legislation?

More informative disclosure of the cost of credit.

RESPA had three specific objectives. What is NOT one of those objectives?

More informative of the cost of credit

Which mortgage would result in higher total payments? Mortgage A: $1,035 a month for 30 years. Mortgage B: $880 a month for 3 years and $1,116 for 27 years

Mortgage A: Total payments = Monthly payment × 12 × Number of years = $1,035 × 12 × 30 = $372,600 Mortgage B: Total payments = (Monthly payment1 × 12 × Number of years1) + (Monthly payment2 × 12 × Number of years2) = ($880 × 12 × 3) + ($1,116 × 12 × 27) = $393,264

Wall street created a new vehicle, the CDO for these professional money managers hungering for a fixed rate investment. The CDO's were made up of pools of:

Mortgage Backed Securities

What would be the net present value of a microwave oven that costs $175 and will save you $84 a year in time and food away from home? Assume an average return on your savings of 5 percent for 6 years. (Hint: Calculate the present value of the annual savings, then subtract the cost of the microwave.)

Net present value = (Annual savings × Present value annuity factor) - Initial cost = ($84 × 5.076) - $175 = $251.38

Which one of the following correctly describes the dividend yield?

Next year's annual dividend divided by today's stock price.

NINA Stands for:

No Income No Asset

Stewart is a fellow finance student at your school who is addicted to day trading and thus buys and sells stocks all day. He never has time to analyze a security so just trades the stock symbols that other investors appear to be trading. Stewart is which one of the following?

Noise Trader

Vandermark Credit Corp. wants to earn an effective annual return on its consumer loans of 17.5 percent per year. The bank uses daily compounding on its loans. What interest rate is the bank required by law to report to potential borrowers?

Nom(17.5 , 365 = 16.13

The term structure of interest rates represents the relationship between which of the following?

Nominal rates on default-free, pure discount bonds and time to maturity

One of your customers is delinquent on his accounts payable balance. You've mutually agreed to a repayment schedule of $600 per month. You will charge 1.0 percent per month interest on the overdue balance. If the current balance is $14,820, how long will it take for the account to be paid off?

Number of months 28.51 ± 1%

Kelly and Tim Jarowski plan to refinance their mortgage to obtain a lower interest rate. They will reduce their mortgage payments by $56 a month. Their closing costs for refinancing will be $1,720. How long will it take them to cover the cost of refinancing?

Number of months = Closing costs / Monthly payment reduction = $1,720 / $56 = 30.71, or 31 months

Mahalo Tours currently has $10,500 in cash. The company owes $26,900 to suppliers for merchandise and $47,500 to the bank for a long-term loan. Customers owe the company $33,000 for their purchases. The inventory has a book value of $62,400 and an estimated market value of $65,600. If the store compiled a balance sheet as of today, what would be the book value of the current assets?

O $100,700 O $79,500 O $85,700 O $105,900*** O $117,500 Current assets = $10,500 + 33,000 + 62,400 - $105,900

British Metals is reviewing its current accounts to determine how a proposed project might affect the account balances. The firm estimates the project will initially require $81,000 in additional current assets and $57,000 in additional current liabilities. The firm also estimates the project will require an additional $8,000 a year in current assets in each of the first three of the four years of the project. How much net working capital will the firm recoup at the end of the project assuming that all net working capital can be recaptured?

O $105,000 O $24,000 $48,000** O $68,000 O $81,000 Net working capital recovery = $81,000 - 57,000 + (3 x $8,000) = $48,000

Healthy Foods just paid its annual dividend of $1.62 a share. The firm recently announced that all future dividends will be increased by 2.1 percent annually. What is one share of this stock worth to you if you require a rate of return of 15.7 percent?

O $11.91 O $12.95 O $12.16 O $10.54 O $13.07 ($1 . 62 x 1 . 021) / ( . 157 - . 021) = $12 . 16

Sherpa Outfitters sells specialty equipment for mountain climbers. Its sales for last year included $488,500 of tents and $800,000 of climbing gear. For next year, management has decided to sell specialty sleeping bags also. As a result of this change, sales projections for next year are $537,350 of tents, $880,000 of climbing gear, and $150,000 of sleeping bags. How much of next year's sales are derived from the side effects of adding the new product to its sales offerings?

O $145,650 -..> O $128,850 O $278,850 O $256,850 Side effects = ($537,350 + 880,000) - ($488,500 + 800,000) - $128,850

Michael's Bakery had $236,400 in net fixed assets at the beginning of the year. During the year, the company purchased $53,200 in new equipment. It also sold, at a price of $22,000, some old equipment that had a book value of $5,900. The depreciation expense for the year was $13,400. What is the net fixed asset balance at the end of the year?

O $260,000 O $283,700 O $276,200 -..> O $270,300 O $289,600 Ending net fixed assets = $236,400 + 53,200 - 5,900 - 13,400 = $270,300

Lew's Auto Repair has cash of $18,600, accounts receivable of $34,500, accounts payable of $28,900, inventory of $97,800, long-term debt of $142,000, and net fixed assets of $363,800. The firm estimates that if it wanted to cease operations today it could sell the inventory for $85,000 and the fixed assets for $349,000. The firm could collect 100 percent of its receivables as they are secured. What is the market value of the firm's assets?

O $332,800 O $458,200 O $374,200 O $495,500 -..> O $487,100 Market value = $18,600 + 34,500 + 85,000 + 349,000 = $487,100

AV Sales has net revenue of $513,000 and costs of $406,800. The depreciation expense is $43,800,interest paid is $11,200, and dividends for the year are$4,500. The tax rate is 33 percent. What is the addition to retained earnings?

O $38,804 O $34,304 O $28,120 O $29,804 O $30,450 Addition to retained earnings = [($513,OOO - 406,800 - 43,800 - - .33)] - $4,500 = Addition to retained earnings = $29,804

Thorkfeld Company incurred depreciation expenses of $28,900 last year. The sales were $755,000 and the addition to retained earnings was $10,200. The firm paid interest of $6,200 and dividends of $5,000. The tax rate was 33 percent. What was the amount of the costs incurred by the company?

O $691,013 O $707,413 O $704,700 O $697,213 O $719,900 Earnings before interest and taxes = [($5,000 + 10,200) / (1 — .33)] + $6,200 = $28,886.57 Costs = $755,000 - 28,900 - 28,8863.57 = $697,213.43

Holly Farms has sales of $509,600, costs of $448,150, depreciation expense of $36,100, and interest paid of $12,400. The tax rate is 28 percent. How much net income did the firm earn for the period?

O $7,778 O $9,324 O $10,380 O $8,671 O $5,886 Net income = ($509,600 - 448,150 - 36,100 - - .28) = $9,324

Taylor Industries has current liabilities of $54,900 and accounts receivable of $88,700. The firm has total assets of $395,000 and net fixed assets of $265,100. The owners' equity has a book value of $147,500. What is the amount of the net working capital?

O $77,400 O $75,000** O $33,800 O -$8,500 O -$2,400 Net working capital = $395,000 - 265,100 - 54,900 = $75,000

Southern Style Realty has total assets of $485,390, net fixed assets of $250,000, current liabilities of $23,456, and long-term liabilities of $148,000. What is the total debt ratio?

O .30 .35 0 69 0 53 0 68 Total debt ratio = ($23,456 + 148,000) / $485,390 = .35

Philippe Organic Farms has total assets of $689,400, long-term debt of $198,375, total equity of $364.182, net fixed assets of $512,100, and sales of $1,021,500. The profit margin is 6.2 percent. What is the current ratio?

O .95 0 1.12 0 1.26 0 1.40*** 0 1.50 Current ratio = ($689,400 — 512,100) / ($689,400 - 364,182 - 198,375) = 1.40

Which of the following is the equation for estimating operating cash flows using the tax shield approach?

OCF = (Sales - Costs) × (1 - Tax rate) + Depreciation × Tax rate

RESPA requires lenders to disclose to buyers a uniform settlement detailing all closing costs within:

One day before the real estate closing

What is the annual opportunity cost of a checking account that requires a $160 minimum balance to avoid service charges? Assume an interest rate of 3 percent.

Opportunity cost = Required minimum balance × Interest rate = $160 × 0.03 = $4.80

Which function in the management process is being utilized when a firm elects to move towards a more centralized structure?

Organizing

In general, which fees have doubled or tripled in price at some financial institutions for checking accounts?

Overdraft fees. Stop-payment orders. Check printing. ALL OF ABOVE

Trenton wants to buy a house but can provide only a 10% down payment. He probably will be required to have

PMI

Maybepay Life Insurance Co. is selling a perpetual annuity contract that pays $2,900 monthly. The contract currently sells for $327,000. What is the monthly return on this investment vehicle?

PV = C / r $327,000 = $2,900 / r We can now solve for the interest rate as follows: r = $2,900 / $327,000 r = .0089, or .89% per month

Which of the following is often considered to offer the least expensive loans (loans with low interest)?

Parents or family members.

This morning, you purchased a stock that will pay an annual dividend of $1.90 per share next year. You require a 12 percent rate of return and the annual dividend increases at 3.5 percent annually. What will your capital gain be on this stock if you sell it three years from now?

Po= $1.90 / (.12-.035) = $22.35 Po3= [$1.90 x (1.0350^3] / (.12 - .035) capital gain = $24.78 - $22.35 = $2.43

Suzie owns five different bonds valued at $36,000 and twelve different stocks valued at $82,500 total. Which one of the following terms most applies to Suzie's investments?

Portfolio

All else equal, an increase in the discount rate decreases ? and increases ? of an annuity.

Present Value and Future Value (in that order)

Katlyn needs to invest $5,318 today in order for her savings account to be worth $8,000 six years from now. Which one of the following terms refers to the $5,318?

Present value

In 2015, a running back signed a contract worth $57 million. The contract called for $10.5 million immediately and a salary of $2.6 million in 2015, $7.7 million in 2016, $10.5 million in 2017, $8.5 million in 2018 and 2019, and $8.7 million in 2020. If the appropriate interest rate is 8 percent, what kind of deal did the running back scamper off with? Assume all payments other than the first $10.5 million are paid at the end of the contract year.

Present value $ 45,359,341.53 ± .01% PV = $10,500,000 + $2,600,000 / 1.08 + $7,700,000 / 1.082 + $10,500,000 / 1.083 + $8,500,000 / 1.084 + $8,500,000 / 1.085 + $8,700,000 / 1.086 PV = $45,359,341.53

An investment offers $8,600 per year for 17 years, with the first payment occurring one year from now. Assume the required return is 9 percent. What would the value be if the payments occurred for 42 years?

Present value $ 92,994.95 ± 0.1%

An investment offers $8,600 per year for 17 years, with the first payment occurring one year from now. Assume the required return is 9 percent. What would the value be if the payments occurred for 77 years?

Present value $ 95,430.12 ± 0.1%

An investment offers $8,600 per year for 17 years, with the first payment occurring one year from now. Assume the required return is 9 percent. What would the value be if the payments occurred forever?

Present value $ 95,555.56 ± 0.1%

An investment offers $8,600 per year for 17 years, with the first payment occurring one year from now. Assume the required return is 9 percent. What is the value of the investment today?

Present value $73,475.23 ± 0.1%

What is the process for converting government enterprises into individually owned firms known as?

Privatization

Advantages of Mutual Funds

Professional Management, low cost, diversification, low minimum investment, flexible & easily tailored to your needs.

Ratios that measure how efficiently a firm manages its assets and operations to generate net income are referred to as _____ ratios.

Profitability

Offshore Drilling Products, Inc., imposes a payback cutoff of three years for its international investment projects. Assume the company has the following two projects available. Year Cash Flow (A) Cash Flow (B) 0 -$45,000 -$ 90,000 1 17,000 19,000 2 23,000 24,000 3 19,000 35,000 4 5,000 250,000

Project A has cash flows of: Cash flows = $17,000 + 23,000 Cash flows = $40,000 during the first two years. The cash flows are still short by $5,000 of recapturing the initial investment, so the payback for Project A is: Payback = 2 + ($5,000 / $19,000) Payback = 2.26 years Project B has cash flows of: Cash flows = $19,000 + 24,000 + 35,000 Cash flows = $78,000 during the first three years. The cash flows are still short by $12,000 of recapturing the initial investment, so the payback for Project B is: Payback = 3 + ($12,000 / $250,000) Payback = 3.05 years

Consider the following four capital budgeting projects. Based on this information, which project does management feel has the highest level of risk? (Note -- no calculations are necessary...this is a conceptual question) Project A Project B Project C Project D CF0 -25,000 -25,000 -50,000 -50,000 CF1 1,000 14,000 5,000 18,000 CF2 2,000 10,000 10,000 18,000 CF3 3,000 6,000 30,000 18,000 CF4 40,000 6,000 40,000 18,000 T 3 years 2.75 years 2.75 years 2.5 years k 10% 12% 12% 14%

Project D

Independent (stand-alone)

Projects are any set of projects in which choosing one has no impact on our decision to choose another project from that set. Taking one project does not influence the other, so they are independent. Decision rule should be based on if the project is good or bad.

Mutually Exclusive

Projects are any set of projects in which choosing one makes the other projects no longer possible. When we have M.E. projects, our decision rule needs to not only decide if a project is good or bad, but needs to be able to rank which project is the best.

Individual Retirement Accounts (IRA)

Provides a tax-incentive for personal retirement savings. Theoretically this should increase the savings rate and encourage more people to prepare for their financial future.

The constant dividend growth model is?

Pt = Dt × (1 + g) / (R - g)

A work-at-home opportunity is available in which you will receive 2 percent of the sales for customers you refer to the company. The cost of your "franchise fee" is $710. How much would your customers have to buy to cover the cost of this fee?

Purchase requirement = Franchise fee / Discount rate = $710 / 0.02 = $35,500

Which of the following is a valid reason for using credit?

Purchasing a car so a homemaker can return to work Borrowing for a college education Purchasing an item that costs less now than it will later Paying for a medical emergency ALL OF ABOVE

In March 2015, Daniela Motor Financing (DMF), offered some securities for sale to the public. Under the terms of the deal, DMF promised to repay the owner of one of these securities $5,000 in March 2050, but investors would receive nothing until then. Investors paid DMF $1,500 for each of these securities; so they gave up $1,500 in March 2015, for the promise of a $5,000 payment 35 years later. a. Assuming you purchased the bond for $1,500, what rate of return would you earn if you held the bond for 35 years until it matured with a value $5,000?

Rate of return 3.49 %

The next dividend payment by Dizzle, Inc., will be $3.05 per share. The dividends are anticipated to maintain a growth rate of 6 percent, forever. Assume the stock currently sells for $49.70 per share. What is the dividend yield? What is the expected capital gains yield?

The dividend yield is the dividend next year divided by the current price, so the dividend yield is: Dividend yield = D1 / P0 Dividend yield = $3.05 / $49.70 Dividend yield = .0614, or 6.14% The capital gains yield, or percentage increase in the stock price, is the same as the dividend growth rate, so: Capital gains yield = 6%

the sensitivity of the bond's price to interest rate changes

The degree of interest rate risk depends on ____.

What would be the net annual cost of the following checking accounts? a. Monthly fee, $3.50; processing fee, 30 cents per check; assume checks written average to 15 per month. (Round your answer to 2 decimal places.) b. Interest earnings of 3 percent with a $450 minimum balance; average monthly balance, $900; monthly service charge of $11 for falling below the minimum balance, which occurs five times a year (no interest earned in these months)

a. Net annual cost = 12 × [(Average number of checks per month × Cost per check) + Monthly fee] = 12 × [(15 × $0.30) + $3.50] = $96.00 b. Net annual cost = Service charges - Interest earnings = (5 × $11) - (7 / 12)(0.03 × $900) = $39.25

Assume a person saves $66 a month by using coupons and doing comparison shopping. a. What is the amount of annual savings? b. What would be the future value of this annual amount over 10 years, assuming an interest rate of 4 percent? Use Exhibit 1-B.

a.Annual savings = Monthly savings × 12 = $66 × 12 = $792 b. Future value = Annual savings × Future value annuity factor = $792 × 12.006 = $9,508.75

Robert Sampson owns a $191,000 townhouse and still has an unpaid mortgage of $156,000. In addition to his mortgage, he has the following liabilities: Visa $ 785 MasterCard 440 Discover card 585 Education loan 3,100 Personal bank loan 500 Auto loan 5,500 Total $ 10,910 Robert's net worth (not including his home) is about $45,000. This equity is in mutual funds, an automobile, a coin collection, furniture, and other personal property. a. What is Robert's debt-to-equity ratio? (Round your answer to 2 decimal places.) b. Has he reached the upper limit of debt obligations?

a.Debt-to-equity ratio = Total debt excluding mortgage / Net worth excluding home = $10,910 / $45,000 = 0.24 b.The upper limit of the debt-to-equity ratio is 1, so he has not reached his upper limit.

Calculate total annual operating cost of the motor vehicle. A. Total variable cost Total fixed cost Total annual operating cost b. Calculate operating cost per mile.

a.Total variable costs = [(Annual mileage / Miles per gallon) × Gas price per gallon] + Oil changes and repairs + Parking and tolls = [(15,240 / 24) × $4.00] + $880 + $760 = $4,180 Total fixed cost = Depreciation + Loan interest + Insurance + License and registration fees = $3,000 + $730 + $935 + $150 = $4,815 Total annual operating costs = Total variable costs + Fixed ownership costs = $4,180 + $4,815 = $8,995 b. Operating cost per mile = Total annual operating costs / Annual mileage = $8,995 / 15,240 = $0.590, or 59.0 cents per mile

Calculate the unit price of each of the following items: (Enter your answers in cents rounded to 2 decimal places.) a. Motor oil—2.0 quarts for $1.60 b. Cereal—15 ounces for $2.25 c. Canned fruit—13 ounces for 90 cents d. Facial tissue—300 tissues for $2.30

a.Unit price = $1.60 / 2.0 quarts = $0.8000, or 80.00 cents per quart b. Unit price = $2.25 / 15 ounces = $0.1500, or 15.00 cents per ounce c. Unit price = $0.90 / 13 ounces = $.0692, or 6.92 cents per ounce d.Unit price = $2.30 / (3,00 tissues / 100) = $0.7667, or 76.67 cents per 100 tissues

We underestimate NPV because of the option(s) to __

abandon; expand

one advantage of the corporate form of organization is the:

ability to raise larger sums of equity capital than other organizational forms.

Cash flows should always be considered on a(n) ___________ basis.

after-tax

The potential conflict of interest between a firm's owners and its managers is referred to as which type of conflict?

agency

call provision

allows issuer to refund the bond issue if rates decline(helps the issuer , but hurts investor), bond investors require higher yields on callable bonds

Short term notes

amounts borrowed from lenders, usually financial institutions such as banks

NASDAQ

an American stock exchange

The shareholders of Weil's Markets would benefit if the firm were to be acquired by Better Foods. However, Weil's board of directors rejects the acquisition offer. This is an example of:

an agency conflict

Form 10K

an annual report required by the Securities and Exchange Commission that provides such information as a letter from the ceo, the firm's history, audited financial statements, management's analysis of the company's performance and executive compensation

Corporation

an entity that legally functions separate and apart from its owners

Venture capitalist

an investment firm or individual that provides money to business start-ups

Lee pays 1 percent per month interest on his credit card account. When his monthly rate is multiplied by 12, the resulting answer is referred to as the:

annual percentage rate.

The return earned in an average year over a multi-year period is called the _____ average return.

arithmetic

The goals of risk analysis in capital budgeting include:

assessing the degree of financing risk identifying critical components

Fixed assets

assets such as equipment, buildings and land

risk aversion

assumes investors dislike risk and require higher rates of return to encourage them to hold riskier securities.

bond value over time

at maturity , the value of any bond must equal its par value.

Smiling Elephant, Inc., has an issue of preferred stock outstanding that pays a $5.40 dividend every year, in perpetuity. If this issue currently sells for $80.30 per share, what is the required return?

The price of a share of preferred stock is the dividend divided by the required return. This is the same equation as the constant growth model, with a dividend growth rate of zero percent. Remember, most preferred stock pays a fixed dividend, so the growth rate is zero. This is a special case of the dividend growth model where the growth rate is zero, or the level perpetuity equation. Using this equation, we find the price per share of the preferred stock is: R = D / P0 R = $5.40 / $80.30 R = .0672, or 6.72%

Capital Budgeting

The process of deciding which long-term projects the firm should undertake.

Total stockholders' equity

common equity in the balance sheet, which is the sum of par value, paid in capital, retained earnings and less any treasury stock

Tomas earned $89 in interest on his savings account last year and has decided to leave the $89 in his account this coming year so it will earn interest. This process of earning interest on prior interest earnings is called:

compounding.

Managerial options are taken into consideration in ____ planning.

contingency

Primary market

corporation receives money by selling new securities, often to an investment banker

Variable costs

costs that change as the quantity of output changes

A bond's annual interest divided by its face value is referred to as the:

coupon rate.

Liquidity preference theory

the theory that the shape of the term structure of interest rates is determined by an investors additional required interest rate in compensation for additional risk

Accounts payable

debt owed to suppliers when firm purchases inventory on credit

Incremental cash flows come about as a(n) ________ consequence of taking a project under consideration.

direct

factors affecting default and bond rating

final performance -Debt ratio -TIE ratio -current ratio Qualitative factors : Bond contract terms -secured vs. unsecured debt -senior vs. subordinated debt -Guarantee and sinking fund provisions -debt maturity

Dry Dock Marina is expected to pay an annual dividend of $1.58 next year. The stock is selling for $18.53 a share and has a total return of 9.48 percent. What is the dividend growth rate?

g= .0948-($1.58/$18.53)=.0095, or .95 percent

Gross profit margin

gross profits divided by sales

Sunk costs are costs that ____.

have already occurred and are not affected by accepting or rejecting a project

Asset management

how efficiently management is using the firm's assets to generate sales

the primary goal of financial management is most associated with increasing the:

market value of the firm

Spot market (cash market)

market where something sells today

Futures market

market where you can buy/sell something at some future date

Operating expenses

marketing and selling expenses, general administrative expenses and depreciation expense

The Treasury yield curve plots the yields on Treasury notes and bonds relative to the ____ of those securities.

maturity

what is the primary goal of financial management for a sole proprietorship?

maximize the market value of the equity

convertible bonds

may be exchanged for common stock of the firm, at the holder's option

Which one of the following is the formula that explains the relationship between the expected return on a security and the level of that security's systematic risk?

capital asset pricing model

According to the radio program, from 2000 to 2006 housing prices were _____ and income was _______.

rising quickly, remaining flat

The expected risk premium on a stock is equal to the expected return on the stock minus the:

risk-free rate.

The U.S. Securities and Exchange Commission periodically charges individuals with insider trading and claims those individuals have made unfair profits. Given this, you would be most apt to argue that the markets are less than _____ form efficient.

strong

Capital Asset Pricing Model(CAPM)

suggests that there is security market line(SML) that states that a stock's required return equals the risk-free return plus a risk premium that reflects the stock's risk after diversification. In conclusion the relevant risk of a stock is its contribution to the riskiness of a well-diversified portfolio.

Which one of the following is a risk that applies to most securities?

systematic

Efficient financial markets fluctuate continuously because:

the markets are continually reacting to new information.

Underwriting

the purchase and subsequent resale of a new security issues. Risk of selling the new issue is assumed by the investment banker

Angela has just received an insurance settlement of $22,500. She wants to save this money until her daughter goes to college. If she can earn an average of 4.7 percent, compounded annually, how much will she have saved when her daughter enters college 6 years from now?

$29,638.94

The global pool of money doubled to what size?

$70 Trillion

AB Industries is an all-equity firm that has $10 per share in cash and a book value per share of $12. At which one of the following market prices would you know with absolute certainty that the stock was mispriced?

$9

Internal Rate of Return

-Calculates the ROR that we can earn on our project. -Acceptance rule is if projects are independent, choose all projects where the IRR is above the required return for those projects. If projects are mutually exclusive, choose the one with the highest IRR. -If Independent: Accept all projects where IRR > k (Required return) -If M.E.: accept project with highest IRR> k

SD of a Portfolio

-Depends not only on the SD and weightings of each stock, but also on the correlation btw. pairs of stocks. -The CORRELATION btw. a pair of stocks measures how closely the returns of each stock are related. A (-) correlation means that the price of 1 stock tends to fall while the other rises (prices/returns are inversely related). A (+) correlation means that the price of 1 stock tends to rise while the other rises (prices/returns are positively related) -Correlations can range from -1.0 to 1.0 -52 of 66 of correlations btw. pairs of stocks are +

A firm has net income of $28,740, depreciation of 6,170, taxes of $13,420, and interest paid of $2,605. What is the cash coverage ratio?

0 8.78 0 20.10 0 14.14 0 16.32 0 19.55 Cash coverage ratio = ($28,740 + 13,420 + 2,605 + 6,170) / $2,605 - 19.55

Lowest Effective Rent

0, 13, 14, 15 ,16

The rate of return on the common stock of Lancaster Woolens is expected to be 21 percent in a boom economy, 11 percent in a normal economy, and only 3 percent in a recessionary economy. The probabilities of these economic states are 10 percent for a boom, 70 percent for a normal economy, and 20 percent for a recession. What is the variance of the returns on this common stock?

0.002244

Priority of Claims in liquidation

1) secured creditors from sales of secured assets 2) trustee's cost 3) wages subject to limits4) taxes 5) unfunded pension liabilities 6) unsecured creditors 7) preferred stock 8) common stock

If a firm's sales estimate used in its base case analysis is 1,000 units per year and they anticipate the upper and lower bounds to be +/- 15%, What is the "best case" for units sold per year?

1,150

Which one of the following bonds is the most sensitive to changes in market interest rates?

10-year, zero coupon

If a new project requires an investment in net working capital when it is launched, then at the end of the project, NWC will be

100% reversed

Suppose that in 2015, a $10 silver certificate from 1898 sold for $11,700. For this to have been true, what would the annual increase in the value of the certificate have been?

6.22 percent

Geometric average return?

= [(1 + R1) × (1 + R2) × ... × (1 + RT)]1/T - 1

- is usually not the same as a bond's coupon rate - changes over time

A corporate bond's yield to maturity:

unsecured

A debenture is a(n) _____ bond, for which no specific pledge of property is made.

Which one of the following will increase the present value of a lump sum future amount to be received in 15 years?

A decrease in the interest rate

Which of the following is an example of open-end credit?

A department store credit card

Which one of the following statements is true?

A discount bond has a coupon rate that is less than the bond's yield to maturity.

-interest is tax deductible -dividends are not tax deductible

A key difference between interest payments and dividend payments is?

A drawback of a regular savings account is

A low rate of return.

$60 in interest at the end of each year for 10 years and a $1,000 repayment of principal at the end of 10 years.

ABC Co. issued 1 million 6 percent annual coupon bonds that mature in 10 years. The face value is $1,000 per bond. What are the expected cash flows from one of these bonds?

You have just made your first $4,000 contribution to your individual retirement account. Assume you earn an annual return 11.35 percent and make no additional contributions. What if you wait 10 years before contributing?

Account value $ 172,273.20

What does an appraisal tell about the value of a property?

An appraisal estimates value

Which of the following usually offers money market funds?

An investment company

Changes in interest rates cause changes in bond prices.

As an investor in the bond market, why should you be concerned about changes in interest rates?

Which of the following can result from a failure to repay a loan?

Bankruptcy. Loss of income or valuable property. Loss of a good reputation. Damage to family relationships. ALL OF ABOVE

which of the following situations is most apt to create an agency conflict?

Basing management bonuses on the length of employment.

What category of ethics relates to matters such as hiring, firing, working conditions, privacy and respect?

Behavior toward employees

Systematic risk is measured by:

Beta

Which one of the following measures the amount of systematic risk present in a particular risky asset relative to the systematic risk present in an average risky asset?

Beta

In March 2015, Daniela Motor Financing (DMF), offered some securities for sale to the public. Under the terms of the deal, DMF promised to repay the owner of one of these securities $5,000 in March 2050, but investors would receive nothing until then. Investors paid DMF $1,500 for each of these securities; so they gave up $1,500 in March 2015, for the promise of a $5,000 payment 35 years later. b. Suppose under the terms of the bond you could redeem the bond in 2022. DMF agreed to pay an annual interest rate of 1.2 percent until that date. How much would the bond be worth at that time?

Bond value $ 1,630.62

Unsystematic risk:

Can be effectively eliminated by portfolio diversification.

Which one of the following is the formula that explains the relationship between the expected return on a security and the level of that security's systematic risk?

Capital Asset Pricing Model

Internal Rate of Return Example

Cash flow= -2000, CF1= 900, CF2= 900, CF3= 900 irr(-2000,{900,900,900}SOLVE = 16.65%

Which of the following are considered relevant cash flows?

Cash flows from beneficial spillover effects Cash flows from erosion effects Cash flows from external costs

What legal document conveys title from one person to another?

Deed

What can be used to determine relationships among different levels of demand and supply, based on price?

Demand and supply curves

What is likely the single MOST effective step that a company can take to set ethical standards?

Demonstrate support from top management

Which description defines the planning function of the management process?

Determining what an organization needs to do and how best to get it done

Suppose a stock had an initial price of $72 per share, paid a dividend of $1.20 per share during the year, and had an ending share price of $79. What was the dividend yield and the capital gains yield?

Dividend Yield 1.67% Capital Gains Yield 9.72%

When evaluating a decision based on the ethical norm of rights, a manager is MOST likely to consider which of the following questions?

Does the decision respect the individuals involved?

What is the environment where a firm conducts business known as?

Domestic business environment

A borrower is considering refinancing and finds that the return, considering refinancing loan payments, is 10%. The borrower can earn 12% on alternative investments so the property...

FALSE

A location quotient is the ratio of total employment to base employment

FALSE

When using the Payback Period as a capital budgeting rule, we should increase the critical acceptance level (T) when evaluating projects with above average risk.

False

Joshua borrowed $2,300 on January 1, 2014, and paid $138 in interest. The bank charged him a service charge of $15. He paid it all back at once on December 31, 2014. What was the APR?

Finance charge = Interest + Service charge = $138 + $15 = $153 APR = Finance charge / Principal = $153 / $2,300 = 0.067, or 6.7%

which one of the following is a working capital decision?

How much cash should the firm keep in reserve?

The dealer's cost is also known as the

Invoice price.

Which of the following is TRUE regarding the incremental cost of borrowing? (A) It should be less than the rate for a first mortgage (B) It should be compared to the cost of obtaining a second mortgage (C) It is used to calculate the APR for the loan (D) It is independent of loan-to-value ratio

It should be compared to the cost of obtaining a second mortgage

Bed Bug Inn has annual sales of $137,000. Earnings before interest and taxes are equal to 5.8 percent of sales. For the period, the firm paid $4,700 in interest. What is the profit margin if the tax rate is 34 percent?

O ¯2.43 percent 0 1.56 percent **** 0 3.33 percent O -5.29 percent O ¯6.11 percent Profit margin = ([(.058 x $137,000) - $4,700] x (1 - .34)) / $137,000 = .0156, or 1.56 percent

What occurs when the quantity demanded exceeds the quantity supplied?

Shortage

With a reverse annuity mortgage the borrower receives payments from the bank

TRUE

Prospectus

Tells you all the important information you need to know. It discusses the investment style of the fund, the risk level of the fund, the various costs of the funds and much more.

The appraised value of a property usually represents:

The actual opinion of an appraiser

Expense Ratio

The amount of money taken out on an annual basis to cover the funds operating expenses. Ranges from .05% at the low end to over 2% at the high end.

Which one of the following best defines the variance of an investment's annual returns over a number of years?

The average squared difference between the actual returns and the arithmetic average return

Which environment is NOT an important dimension of a business organization external environment?

The corporate cultural environment

adjustable

The coupon payments on floating-rate bonds are _____.

annuity present value

To find the total bond value, add the present value of the amount paid at maturity to the _____ of the annual coupon payments.

Tobin's Q relates the market value of a firm's assets to which one of the following?

Today's cost to duplicate those assets

False

True or false: A debenture is a bond secured with collateral.

true

True or false: A put bond allows the holder to force the issuer to buy the bond back at a stated price.

True

True or false: Current yield = Annual coupon payment/Price

true

True or false: Investors require a premium for the risk that issuers other than the Treasury may not make all promised payments on the issued bonds.

False

True or false: The higher the coupon rate, the greater the interest rate risk, all other things being equal.

The next dividend payment by Dizzle, Inc., will be $2.65 per share. The dividends are anticipated to maintain a growth rate of 6.50 percent, forever. If the stock currently sells for $48.90 per share, what is the required return?

We need to find the required return of the stock. Using the constant growth model, we can solve the equation for R. Doing so, we find: R = (D1 / P0) + g R = ($2.65 / $48.90) + .0650 R = .1192, or 11.92%

-The repayment arrangements -The total amount of bonds issued

Which of the following are usually included in a bond's indenture?

Equity is publicly traded while debt is not

Which of the following is not a difference between debt and equity?

Market Risk premium

additional return over the risk-free rate needed to compensate investors for assuming an average amount of risk, its size depends on the perceived risk of the stock market and investors' degree of risk aversion, varies from year to year but most estimates suggest that it ranges between 4% and 8% per year.

Capital market

all institutions and procedures that facilitate transactions in long-term financial instruments

General partnership

all partners are fully liable for the in debtness incurred by the partnership

Over the-counter market

all security markets except organized security exchanges. A network of brokers and dealers linked by computer

putable bond

allows holder to sell the bond back to the company prior to maturity

Ricky has a conventional mortgage. He can monitor the reduction of his loan balance through his payments by using _____ information.

amortization

Probably the least effective means of aligning management goals with shareholder interests is:

automatically increasing management salaries on an annul basis.

Which one of the following measures the amount of systematic risk present in a particular risky asset relative to the systematic risk present in an average risky asset?

beta

The price at which a dealer will purchase a bond is referred to as the _____ price.

bid

A bond trader just purchased and resold a bond. The amount of profit earned by the trader from this purchase and resale is referred to as the:

bid-ask spread.

The call premium is the amount by which the:

call price exceeds the par value.

How sinking funds executed

call x% of the issue at par, for sinking fund purposes, likely to be used if (rd) is below the coupon rate and the bond sells at a premium. If you buy bonds in the open market its likely to be used if (rd) is above the coupon rate and the bond sells at a discount.

A positive NPV exists when the market value of a project exceeds its cost. Unfortunately, most of the time the market value of a project:

cannot be observed

Jenna has been promoted and is now in charge of all external financing. In other words, she is in charge of:

capital structure

Uptown Markets is financed with 45 percent debt and 55 percent equity. This mixture of debt and equity is referred to as the firm's:

capital structure

Cash flows used in project estimation should always reflect:

cash flows when they occur; after-tax cash flows

A loan has an APR of 8.5 percent and an EAR of 8.5 percent. Given this, the loan must:

charge interest annually.

The inflation premium:

compensates investors for expected price increases.

Income after deducting vacancy that is available to pay expenses is referred to as

effective gross income

Assume that the market prices of the securities that trade in a particular market fairly reflect the available information related to those securities. Which one of the following terms best defines that market?

efficient capital market

Federal income tax policy has generally been thought to:

encourage homeownership

The possibility that errors in projected cash flows will lead to incorrect decisions is known as:

estimation risk forecasting risk

The number of positive NPV projects is unlimited for any given firm.

false

True or false: Sensitivity analysis is helpful because it indicates what we should do regarding forecasting errors.

false

True or false: Taxes are based on the difference between the initial cost and the sales price.

false

Organized security exchange

formal organization for buying and selling securities

Failure to Diversify

if an investor chooses to hold a one-stock portfolio (doesnt diversify) would the investor be compensated for the extra risk they bear? NO , stand alone risk is not important to a well-diversified investor.rational risk-averse investors are concerned with SDportfolio which is based upon market risk.

An increase in depreciation expense will ____ cash flows from operations.

increase

Synergy will _____ the sales of existing products.

increase

The stand-alone principle assumes that evaluation of a project may be based on the project's ________________ cash flows.

incremental

The difference between a firm's cash flows with a project versus without the project is called ________

incremental cash flows

Secondary market

investors buy and sell existing securities

Which one of the following is an example of systematic risk?

investors panic causing security prices around the globe to fall precipitously

CAPM

investors seem to be concerned with both market risk and total risk,therefore the SML may not produce a correct estimate of Ri . concepts are based upon expectations but beta are calculated using historial data.A company's historial data may not reflect investors expectation about future riskiness.

Common stockholders

investors who own the firm's common stock

a corporation:

is a legal entity separate from its owners.

Collections

is counter cyclical with the economy and has a negative correlation. This is usual

When a bond's yield to maturity is less than the bond's coupon rate, the bond:

is selling at a premium.

reinvestment risk

is the concern that (rd) will fall and future CFs will have to be reinvested at lower rates, hence reducing income,

Debt

liabilities consisting of such sources as credit extended by suppliers or a loan from a bank

A protective covenant

limits the actions of the borrower.

Long term debt

loans from banks or other sources that lend money for longer than 12 months

warrant

long-term option to buy a stated number of share common stock at a specified price

What is not tax deductible for homeowners:

maintenance expenses

The primary risk in estimation errors is the potential to ____

make incorrect capital budgeting decisions

The project cash flow equals the project operating cash flow ______ project change in NWC minus project capital spending.

minus

types of bonds

mortgage bonds, debentures, subordinated debentures, investment-grade bonds, junk bonds.

A mortgage is BEST defined as a legal document that:

names real estate as the security or collateral for the repayment of a loan.

Net profit margin

net income divided by sales

Earnings per share

net income on a per share basis

Which one of the following correctly describes the dividend yield?

next year's annual dividend divided by today's stock price

Which one of the following is defined by its mean and its standard deviation?

normal distribution

SD of a 2-stock Portfolio Equation

op= SQUARE ROOT ON ALL! W12o12 + W22o22 + 2W1W2o1o2corr1,2 op= the SD of portfolio Wi= weight (proportion) of stock 1 o1= the SD of stock 1 W2= weight (proportion) of stock 2 o2= the SD of stock 2 Corr1,2= correlation btw the returns of stocks 1 and 2

What is the term given to the groups, individuals, and organizations that are directly affected by the practices of an organization?

organizational stakeholders

Common sized income statement

place where a firm's expenses and profits are expressed as a % of its sales

market risk

portion of a security's stand alone risk that cannont be eliminated through diversification measured by beta.

sinking fund

provision to pay off a loan over its life rather than all at maturity, similar to amortization on a term loan, reduces risk to investor, shorten average maturity, but not good for investors if rates decline after issuance.

Inventories

raw materials, work in progress, and finished goods held by the firm for eventual sale.

Given a level of investment in net working capital, that same investment must be _____ at some time in the future.

recovered

The supply of space is:

relatively inelastic in the short run, and highly elastic in the long run

One of the most important steps in estimating cash flow is to determine the _________ cash flows.

relevant

West Corporation estimated cash flows for a project, evaluated those cash flows using NPV, and determined that the project was acceptable. Unfortunately West Corporation lost money on the project. This may have been avoided had they assessed the ______ of the cash flow estimates.

reliability

Given an interest rate of zero percent, the future value of a lump sum invested today will always:

remain constant, regardless of the investment time period.

Facts about common stock

represents ownership,ownership implies control, stockholders elect directors,directors elect management, management's goal :maximize the stock price.

The excess return is computed as the:

return on a risky security minus the risk-free rate.

Which one of the following is a positively sloped linear function that is created when expected returns are graphed against security betas?

security market line

Negotiation activities are part of the ____ phase of the research-based buying process.

selection and purchase

____ analysis is useful in pinpointing variables that deserve the most attention.

sensitivity

To investigate the impact on NPV of a change in one variable, you would employ ________.

sensitivity analysis

Competitive bid purchase

several underwriting groups bid for the right to purchase the new issue from the corporation that is raising funds

Liquidity risk premium

the additional return required by investors for securities that cannot be quickly converted into cash at a reasonably predictable price

Maturity risk premium

the additional return required by investors in longer-term securities to compensate them for the greater risk of price fluctuations on those securities caused by interest

Default risk premium

the additional return required by investors to compensate them for the risk of default

Paid in-capital

the amount a company receives above par value

Dividends per share

the amount of dividends a firm pays for each share outstanding.

Par value

the arbitrary value a firm puts on each share of stock prior to its being offered for sale

An agency issue is most apt to develop when:

the control of a firm is separated from the firm's ownership.

Cost of goods sold

the cost of producing or acquiring a product or service to be sold in the ordinary course of business

Net income

the earnings available to the firm's common stockholder

Treasury stock

the firms stock that has been issued and then repurchased by the firm

Initial public offering (IPO)

the first time a company issues its stock to the public

Price/book ratio

the market value of a share of the firm's stock divided by the book value per share of the firm's equity in the balance sheet.

the primary goal of financial management is to maximize:

the market value of existing stock

Capital rationing exists when a company has identified positive NPV projects but cannot (or will not) find:

the necessary financing

Real rate of interest

the nominal (quoted) rate of interest less any loss in purchasing power of the dollar during the time of the investment

The Market Value of a loan is:

the present value of the remaining payment

Seasoned equity offering (SEO)

the sale of additional stock by a company whose shares are already publicly traded

Market segmentation theory

the term structure of interest rates is built on the notion that legal restrictions and personal preferences limit choices for investors to certain ranges of maturities

Book value

the value of an asset as shown on a firm's balance sheet

RESPA requires lenders to disclose to buyers a good faith estimate of certain closing costs within:

three days after loan application

The difference between the existing stock of space and equilibrium occupancy is known as the:

vacancy

Calculating beta

well-diversified investors are primarily concerned with how a stock is expected to move relative to the market in the future, the slope of the regression line is defined as the beta coefficenent for the security.

A credit card has an annual percentage rate of 12.9 percent and charges interest monthly. The effective annual rate on this account:

will be greater than 12.9 percent.

T-Bill

will return the promise 5.5% regardless of the econonmy. No t-bill do not provide a completely risk-free return as they are still exposed to inflation. Also risky in terms of reinvestment risk, are risk-free in the default sense of the word.

the daily financial operations of a firm are primarily controlled by managing the:

working capital

Negative beta

yes if the correlation between stock i and the market is negative, if the correlation is negative the regression line would slope downward , and the beta would be negative. However beta is highly unlikely.

which one of the following is a capital structure decision?

Establishing the preferred debt-equity level

-Default risk premium -Liquidity premium

Which of the following may increase the yield on corporate bonds as compensation to investors but will not impact Treasury bond yields?

Original issue price of bond

Which of the following variables is NOT required to calculate the value of a bond?

Price/earnings ratio

the price the market places on $1 of a firm's earnings

Privileged subscription

the process of marketing a new security issue to a select group of investors

Term structure of interest rates (yield curve)

the relationship between interest rates and the term to maturity, where the risk of default is held constant

Real risk free interest rate

the required rate of return on a fixed income security that has no risk in an economic environment of zero inflation.

Direct sale

the sale of securities by a corporation to the investing public without the services of an investment banker

While Paul was shopping for a new chainsaw, he found a great deal on a new drill and bought it. This is an example of

Impulse shopping.

Marginal tax rate

the tax rate that would be applied to the next dollar of income

You own a stock portfolio invested 25 percent in Stock Q, 15 percent in Stock R, 20 percent in Stock S, and 40 percent in Stock T. The betas for these four stocks are 0.59, 1.66, 1.35, and 0.66, respectively. What is the portfolio beta?

0.93

What is the total number of inputs that change at a given time while doing sensitivity analysis?

1

Capitalization rates will differ from yield rates when the income is expected to

Increase & Decrease. Capitalization rates equal yield rates only when income doesn't change

Unbiased expectations theory

the theory that the shape of the term structure of interest rates is determined by an investors expectations about future interest rates.

Which type of business constituent has titles such as President and CEO?

top managers

You have 6 tenants paying $200/month. Market rents are now $250/month. What is lease you record for each month?

$300 (6 x (250-200))

To prepare proforma financial statements, estimates of quantities such as unit sales, selling price per unit, variable cost per unit, and total fixed costs are required.

true

True or false: Investment in net working capital may arise from the need to cover credit sales.

true

In a competitive market, positive NPV projects are:

uncommon

Suppose a stock had an initial price of $72 per share, paid a dividend of $1.20 per share during the year, and had an ending share price of $79. Compute the percentage total return

11.39%

The risk-free rate of return is 3.9 percent and the market risk premium is 6.2 percent. What is the expected rate of return on a stock with a beta of 1.21?

11.40 percent

Marcus can afford a monthly mortgage payment of $900. If he is eligible for a 30-year, 5% mortgage (where the mortgage factor is 5.37), how much of a mortgage loan can he afford?

$167,597.77 Affordable mortgage amount = (affordable monthly mortgage payment/mortgage factor) × 1,000; thus, = ($900/5.37) × 1,000 = $167,597.77

You would like to establish a trust fund that would provide annual scholarships of $100,000 forever. How much would you have to deposit today in one lump sum to achieve this goal if you can earn a guaranteed 4.5 percent rate of return?

$2,222,222; P = $100,000/.045 = $2,222,222

Broadband, Inc., has estimated preliminary cash flows for a project and found that the NPV for those cash flows is $400,000. The company now plans to perform a scenario analysis on the cash flow and NPV estimates. It will use an NPV of _____ as the base case.

$400,000

At the end of this month, Les will start saving $200 a month for retirement through his company's retirement plan. His employer will contribute an additional $.50 for every $1.00 that he saves. If he is employed by this firm for 30 more years and earns an average of 8.25 percent on his retirement savings, how much will he have in his retirement account 30 years from now?

$470,465.70

The subject of an appraisal has only two bedrooms, but one of the comparables used in the appraisal has 3. If the adjustment for the 3rd bedroom is $5000, the adjustment would be:

$5000 decrease to the comparable's selling price

Today, Stacy is investing $18,000 at 6.72 percent, compounded annually, for 5 years. How much additional income could she earn if she had invested this amount at 7.15 percent, compounded annually?

$506.06

Best's Fried Chicken just took out an interest-only loan of $50,000 for three years with an interest rate of 8.15 percent. Payments are to be made at the end of each year. What is the amount of the payment that will be due at the end of Year 3?

$54,075.00; PaymentYear 3 = $50,000 + ($50,000 x .0815) = $54,075.00

What is the principle portion of the 222 payments of a fully amortizing $250,000, 30 year fixed rate loan with an interest rate of 4.825% a. 720.34 b. 894.95 c. 689.93 d. 753.07

$753.07

A mutual fund may contain ___.

- Common stocks from a variety of industries. - Treasury Bonds - International Stocks

401K Tips

-Dont use the company stock as your main investment -Dont let company stock represent > 20% of your retirement assets. < 10% is ideal.

Non-diversifiable Risk

-Leftover risk -examples include political events, energy price shocks, changes in interest rates, recessions, etc.

Information about a personal bankruptcy may be reported in your credit file for?

10 years.

A stock has a beta of 1.4, the expected return on the market is 10 percent, and the risk-free rate is 4 percent. What must the expected return on this stock be?

12.4%

The rules for depreciating assets for tax purposes are based upon provisions in the:

1986 Tax Reform Act

You are considering the purchase of a new stock. The stock is forecasted to pay a dividend next year (D1) of $4.24. In addition, you forecast that the firm will have a stable growth rate of 2.9% for the foreseeable future. The current risk-free rate of return is 3.1%. The expected return on the market is 12% and the standard deviation for the market is 18%. The stock has a correlation to the market of 0.57. Finally, the stock has a standard deviation of 45%. Given this information, what is the value of this stock?

32.91 The first step is to estimate beta by taking the standard deviation of the stock times the correlation then dividing by the standard deviation of the market. Once you have beta, plug it into the Security Market Line Formula to get required return. Finally, once you have required return, plug it into the stock valuation model.

Rob wants to invest $15,000 for 7 years. Which one of the following rates will provide him with the largest future value?

4 percent interest, compounded annually

You have been told that you need $32,000 today for each $100,000 you want when you retire 28 years from now. What rate of interest was used in the present value computation? Assume interest is compounded annually.

4.15 percent

Wiggle Pools has total equity of $358,200 and net income of $47,500. The debt-equity ratio is .68 and the total asset turnover is 1.2. What is the profit margin

4.82 percent 0 5.23 percent 0 5.67 percent 0 6.58 percent** 0 7.31 percent Profit margin = x (1 + .68)] = .0658, or 6.58 percent

The 6 percent semiannual coupon bonds of IPO, Inc., are selling for $1,087. The bonds have a face value of $1,000 and mature in 11 years. What is the yield to maturity?

4.96 percent

The U.S. Department of Commerce considers a business "small" if it has fewer than how many employees?

500

You are considering the purchase of a new stock. The stock is forecasted to pay a dividend next year (D1) of $4.8. In addition, you forecast that the firm will have a stable growth rate of 2.1% for the foreseeable future. The current risk-free rate of return is 2.7%. The expected return on the market is 8.7% and the standard deviation for the market is 13%. The stock has a correlation to the market of 0.48. Finally, the stock has a standard deviation of 39%. Given this information, what is the value of this stock?

51.95

You are needing $200,000 in financing to buy a home. Two loans are being offered by your bank, both for 30 year terms. $175,000 at 6% $25,000 at 8% What is the effective interest rate for the $200,000 if you accept this package? a. 9% b. 6.26% c. 8.76% d. 7.09%

6.26%

The 6.3 percent, semi-annual coupon bonds of PE Engineers mature in 13 years and have a price quote of 99.2. These bonds have a current yield of _____ percent, a yield to maturity of _____ percent, and an effective annual yield of _____ percent.

6.35; 6.39; 6.49

call provision

A provision in the bond indenture giving the issuing company the option to repurchase the bonds before maturity is termed a _________________.

Which of the following is an important aspect of the loan refinance decision process? (A) Terms associated with the existing loan (B) Terms of the new loan (C) Fees associated with paying off the old loan and/or acquiring the new loan (D) All of the above

All of the above

A class-action suit against a utility company resulted in a settlement of $1.85 million for 68,000 customers. If the legal fees, which must be paid from the settlement, are $400,000, what amount will each plaintiff receive?

Amount per plaintiff = (Settlement amount - Legal fees) / Number of plaintiffs = ($1,850,000 - $400,000) / 68,000 = $21.32

Suppose you are committed to owning a $245,000 Ferrari. If you believe your mutual fund can achieve an annual return of 11.1 percent, and you want to buy the car in 10 years on the day you turn 30, how much must you invest today?

Amount to be invested $ 85,511.69

Which level of management sets general policies, formulates strategies, approves all significant decisions, and represents the company in dealings with other firms and with government bodies?

Top managers

The return earned in an average year over a multi-year period is called the _____ average return

Arithmetic

Calculating Impact of Load Charges and Expense Ratios

Assume you are dealing with a mutual fund that has a 4% sales load and a 1.25% expense ratio. If you invest $10,000 today and $200 per month for 30 years, how much are these costing you if the fund earns 10% before any fees? Find FV N= 360 I= 10 PV= 10,000 PMT= 200 P/Y=12 FV=650,471.58 Calculate FV with fees -1.25% expense ratio and 4% load charge. N= 360 I= 8.75 PV= 9,600 PMT= 192 P/Y=12 FV=$464,984.97

ST Trucking just signed a $3.8 million contract. The contract calls for a payment of $1.1 million today, $1.3 million one year from today, and $1.4 million two years from today. What is this contract worth today at a discount rate of 8.7 percent?

B. $3,480,817.37

Beta Equation

BA= {(oA)(corra,mkt) / omkt} BA= the Beta of Stock A oA= SD of Stock A coorA,mkt= correlation btw Stock A & overall market omkt= SD of overall market

The lowest rating a bond can receive from Standard and Poor's and still be classified as an investment-quality bond is:

BBB.

Graphically, what has been achieved when price of goods demanded is equal to the quantity of goods supplied?

Equilibrium price

Identify the three main sources of cash flows over the life of a typical project.

Cash outflows from investment in plant and equipment at the inception of the project; Net cash flows from salvage value at the end of the project; Net cash flows from sales and expenses over the life of the project

Which of the following are components of project cash flow?

Change in net working capital Operating cash flow Capital spending

Bankrupty(Reorganization)

Company's usually reorganization , if cant meet obligations it files under ch.11 to stop creditors from foreclosing, taking assets, and closing business and it has 120 days to file reorganization plan. court appoints a "trustee" to supervise reorganizations, management usually stays in control. "worth more alive than dead"

What tends to have lower effective rents?

Concessions

The tendency for a decision maker to search for confirmation that a recent decision he or she made was a good decision represents which one of the following characteristics?

Confirmation Bias

When an activity benefits an individual but not the employer, what ethical dilemma has been created?

Conflict of interest

Stock C has an E.R. of 7% and a SD of 20%. Stock D has an E.R. of 9% and a SD of 28%. Which should you choose?

Could choose both. Stock D is more riskier but compensates with a higher E.R.

BDJ Co. wants to issue new 18-year bonds for some much-needed expansion projects. The company currently has 9.9 percent coupon bonds on the market that sell for $1,139, make semiannual payments, have a $1,000 par value, and mature in 18 years. What coupon rate should the company set on its new bonds if it wants them to sell at par?

Coupon rate 8.40%

PK Software has 7.8 percent coupon bonds on the market with 25 years to maturity. The bonds make semiannual payments and currently sell for 108.75 percent of par. What is the current yield on PK's bonds?

Current yield 7.17%

Which description identifies the organizing function of the management process?

Determining how best to arrange an organization's resources into a coherent structure

The first step in the planning process is determining goals. What is the next step?

Developing a comprehensive strategy

The stock price of Jenkins Co. is $53. Investors require a 12 percent rate of return on similar stocks. If the company plans to pay a dividend of $3.15 next year, what growth rate is expected for the company's stock price? Explanation: We need to find the growth rate of dividends. Using the constant growth model, we can solve the equation for g.

Doing so, we find: g = R - (D1 / P0) g = .12 - ($3.15 / $53) g = .0606 or 6.06%

The formula which breaks down the return on equity into three component parts is referred to as which one of the following?

DuPontIdentity

City Motors will sell a $15,000 car for $345 a month for 52 months. What is the interest rate?

E. 8.38 percent

Total possible income less any vacancy is

EGI (effective gross income)

According to the IMF, the Global Pool of Money doubled during which time frame?

Early 2000s

First National Bank charges 10.4 percent compounded monthly on its business loans. First United Bank charges 10.6 percent compounded semiannually. Calculate the EAR for each bank.

Eff(10.4 , 12 = 10.91 & Eff(10.6 , 2 = 10.88

Quinn applied for a loan. He provided information about his finances and the home he plans to purchase. Results of the affordability mortgage calculation included all of the following except

Expected maintenance costs.

The titles of supervisor, office manager, and group leader are examples of which level of management?

First-line management

Which type of manager spends most of their time working with and supervising the employees who report to them?

First-line managers

Which one of the following refers to the fact that an individual may reply differently if a question is asked in a different manner?

FrameDependence

What type of estate lasts for an indefinite period of time?

Freehold Estate

Most products come with some guarantee of quality. This type of warranty states that a defective product can be fixed or replaced during a reasonable amount of time.

Full warranty

ABC Shares

Funds sold with various expense packages. Used to pay the brokers/advisors who sell these shares. A shares have higher front-end load charges. B have higher annual expenses and back-end loads. C shares typically avoid load charges but have higher annual expenses. Shortterm holding use C, longterm holdings used A

Booker, Inc., has identified an investment project with the following cash flows. Year Cash Flow 1 $ 970 2 1,200 3 1,420 4 2,160 If the discount rate is 7 percent, what is the future value of these cash flows in Year 4?

Future value $ 6,241.57 ± 0.1% FV@7% = $970(1.07)^3 + $1,200(1.07)^2 + $1,420(1.07) + $2,160 = $6,241.57

Booker, Inc., has identified an investment project with the following cash flows. Year Cash Flow 1 $ 970 2 1,200 3 1,420 4 2,160 What is the future value at an interest rate of 13 percent?

Future value $ 6,696.49 ± 0.1% FV@13% = $970(1.13)^3 + $1,200(1.13)^2 + $1,420(1.13) + $2,160 = $6,696.49

Booker, Inc., has identified an investment project with the following cash flows. Year Cash Flow 1 $ 970 2 1,200 3 1,420 4 2,160 What is the future value at an interest rate of 22 percent?

Future value $ 7,439.85 ± 0.1% FV@22% = $970(1.22)^3 + $1,200(1.22)^2 + $1,420(1.22) + $2,160 = $7,439.85

Your grandparents just gave you a gift of $6,500. You are investing this money for 6 years at 4 percent simple interest. How much money will you have at the end of the 6 years?

Future value = $6,500 + ($6,500 ×.04 ×6) = $8,060

Which description identifies the controlling function of the management process?

Monitoring a firm's performance to ensure that it is meeting its goals

Diversification

Refers to the concept that by holding a # of different securities from a spectrum of industries, we can negate the impact of company specific factors on our returns.

Which one of the following categories of securities had the highest average return for the period 1926-2010?

Small Company Stocks

Which one of the following categories of securities has had the most volatile returns over the period 1926-2010?

Small Company Stocks

Which of the following is a type of electronic payment?

Smart card

Stock A has an E.R. of 10% and a SD of 25%. Stock B has an E.R. of 10% and a SD of 30%. Which should you choose?

Stock A because Stock B offers no additional compensation.

Interdependent

The decision to take one project impacts our decision to take another, but they are not mutually exclusive. Some interdependent projects are compliments in which the cash flows from both projects takes together are greater than the cash flows from each project on a standalone basis. Other interdependent projects are substitutes in which the cash flows from both projects taken together are less than the cash flows from each project on a standalone basis.

Size Problem

The issue with the size problem is related to IRR's focus on ROR instead of value generation in terms of dollars. If initial investments are vastly different, we need to be aware of the size problem and use NPV if dealing with M.E. projects.

A line of credit is

The maximum dollar amount of credit the lender has made available to a borrower.

Among the three main sources of cash flow, which source of cash flow is the most important and also the most difficult to forecast?

The operating cash flows from net sales over the life of the project

A manager has estimated a positive NPV for a project. What could drive this result?

The project is a good investment. The cash flow estimations are inaccurate. Overly optimistic management

Which is NOT part of the definition of market value?

The property has been on the open market for less than a year

Consider the following two projects: Project A Project B •CF0 -15,000 -15,000 •CF1 10,000 2,000 •CF2 8,000 3,000 •CF3 5,000 22,000 Assuming these projects are mutually exclusive, we could have problems when using IRR due to ___.

The reinvestment rate problem The size problem does not appear here as both projects have the same initial investment. The crossover (multiple IRR) problem does not appear here as both projects only have on crossover point (between year 0 and year 1) so there will only be a single, viable IRR solution for each. However, project A is frontloaded while project B is backloaded which could lead to a Reinvestment Rate Problem.

the Fisher effect

The relationship between nominal rates, real rates and inflation is called ________ .

Mitchell, Inc., is expected to maintain a constant 5.75 percent growth rate in its dividends, indefinitely. If the company has a dividend yield of 4.25 percent, what is the required return on the company's stock?

The required return of a stock is made up of two parts: The dividend yield and the capital gains yield. So, the required return of this stock is: R = Dividend yield + Capital gains yield R = .0425 + .0575 R = .1000, or 10.00%

Mitchell, Inc., is expected to maintain a constant 5.85 percent growth rate in its dividends, indefinitely. If the company has a dividend yield of 4.35 percent, what is the required return on the company's stock?

The required return of a stock is made up of two parts: The dividend yield and the capital gains yield. So, the required return of this stock is: R = Dividend yield + Capital gains yield R = .0435 + .0585 R = .1020, or 10.20%

You bought a share of 6.80 percent preferred stock for $98.68 last year. The market price for your stock is now $103.92. What is your total return for last year?

The return of any asset is the increase in price, plus any dividends or cash flows, all divided by the initial price. Since preferred stock is assumed to have a par value of $100, the dividend was $6.80, so the return for the year was: R = ($103.92 - 98.68 + 6.80) / $98.68 R = .1220, or 12.20%

Suppose a stock had an initial price of $93 per share, paid a dividend of $1.90 per share during the year, and had an ending share price of $111. Compute the percentage total return What was the dividend yield? What was the capital gains yield?

The return of any asset is the increase in price, plus any dividends or cash flows, all divided by the initial price. The return of this stock is: R = [($111 - 93) + 1.90] / $93 R = .2140, or 21.40% The dividend yield is the dividend divided by the initial price, so: Dividend yield = $1.90 / $93 Dividend yield = .0204, or 2.04% And the capital gains yield is the increase in price divided by the initial price, so: Capital gains yield = ($111 - 93) / $93 Capital gains yield = .1935, or 19.35%

Time to Maturity Coupon rate

The sensitivity of a bond's price to interest rate changes is dependent on which of the following two variables?

What document usually summarizes the sources, disbursements, charges and credits association with a real-estate closing?

The settlement statement

-the pure time value of money -the relationship between nominal rates and time to maturity

The term structure of interest rates describes ________.

Who customarily do NOT attend real estate closings?

They buyer's and sellers immediate family

All else equal, an increase in the beta of the stock will lower the stock price.

True

It shows the yield for different maturities of Treasury notes and bonds

What does a Treasury yield curve show?

GAAP

a rule based set of accounting principles, standards and procedures that companies use to compile their financial statements.

Balance sheet

a snapshot of the firm's financial position at a specific time, presenting its assets, liabilities and stockholders

a. What would be the future value of a savings account started with $750, earning 5 percent (compounded annually) after 10 years? b. Brenda Young desires to have $11,000 eight years from now for her daughter's college fund. If she will earn 4 percent (compounded annually) on her money, what amount should she deposit now? Use the present value of a single amount calculation. c. What future value amount would you have if you deposited $2,600 a year for 25 years at 7 percent (compounded annually)?

a. FV = $750 × 1.629 = $1,222 (Exhibit 1-A) b. PV = $11,000 × 0.731 = $8,041 (Exhibit 1-C) c. FV = $2,600 × 63.249 = $164,447 (Exhibit 1-B

After visiting several automobile dealerships, Richard selects the car he wants. He likes its $11,000 price, but financing through the dealer is no bargain. He has $2,200 cash for a down payment, so he needs an $8,800 loan. In shopping at several banks for an installment loan, he learns that interest on most automobile loans is quoted at add-on rates. That is, during the life of the loan, interest is paid on the full amount borrowed even though a portion of the principal has been paid back. Richard borrows $8,800 for a period of four years at an add-on interest rate of 8 percent. a. What is the total interest on Richard's loan? b. What is the total cost of the car? c. What is the monthly payment? d. What is the annual percentage rate (APR)?

a. I = P × r × T = $8,800 × 0.08 × 4 = $2,816 b. Total cost = Down payment + Interest + Principal = $2,200 + $2,816 + $8,800 = $13,816 c. Monthly payment = (Interest + Principal) / Number of months = ($2,816 + $8,800) / (4 × 12) = $242 d. APR = (2 × n × I) / [P(N + 1)] = (2 × 12 × $2,816) / [$8,800($48 + 1)] = 0.1567, or $15.67%

Sidney took a $300 cash advance by using checks linked to her credit card account. The bank charges a 4 percent cash advance fee on the amount borrowed and offers no grace period on cash advances. Sidney paid the balance in full when the bill arrived. a. What was the cash advance fee? b. What was the interest for one month at an APR of 16 percent? c. What was the total amount she paid? d. What if she had made the purchase with her credit card and paid off her bill in full promptly? Assume the credit card has a grace period.

a. Cash advance fee = Cash advance fee percent × Cash advance amount = 0.04 × $300 = $12.00 b. Monthly interest = (Annual rate / 12) × Cash advance amount = (0.16 / 12) × $300 = $4.00 c. Total amount paid = Cash advance fee + Monthly interest + Cash advance amount = $12.00 + $4.00 + $300 = $316.00 d. Total amount paid = Cash advance amount = $300.00 If her credit card did not have a grace period, then she would also have owed $4.00 for monthly interest

Public offering

securities are made available to all individual and institutional investors

What takes place when buyers purchase more of a product when the price drops, and less when the price increases?

Laws of demand

What is the primary force that determines what a firm buys and sells?

Laws of supply and demand

bond

a long-term debt instrument in which a borrower agrees to make payments of principal and interest, on specific dates, to the holders of the bond.

SD

a measure of volatility or total risk, which is appropriate when evaluation stand-alone risk.

Dutch auction

a method of issuing securities by which investors place bids indicating how many shares they are willing to buy and at what price.

Limited partnership

a partnership in which one or more of the partners has limited liability, restricted to the amount of capital he or she needs

The Jones Brothers recently established a trust fund that will provide annual scholarships of $12,000 indefinitely. These annual scholarships are:

a perpetuity.

Mutual Fund

a pooled investment portfolio managed by a professional portfolio manager.

Inflation premium

a premium to compensate for anticipated inflation that is equal to the price change expected to occur over the life of the bond or investment instrument.

You purchased 440 shares of stock at a price of $50.16 per share. Over the last year, you have received total dividend income of $505. What is the dividend yield?

2.3%

According to the radio program, what was the year that credit standards really begun to start fallin in the mortgage business?

2003

The returns on the common stock of New Image Products are quite cyclical. In a boom economy, the stock is expected to return 32 percent in comparison to 14 percent in a normal economy and a negative 28 percent in a recessionary period. The probability of a recession is 25 percent while the probability of a boom is 20 percent. What is the standard deviation of the returns on this stock?

21.41 percent

Consider two stocks. Stock A has a standard deviation of 29% and stock B has a standard deviation of 48%. The stocks have a correlation of -0.13. You plan to invest $6572 into stock A and $8818 into stock B. What is the standard deviation of your two stock portfolio?

28.66

Consider two stocks. Stock A has a standard deviation of 21% and stock B has a standard deviation of 50%. The stocks have a correlation of 0.28. You plan to invest $8919 into stock A and $8642 into stock B. What is the standard deviation of your two stock portfolio?

29.43 (Solve using the 2 stock portfolio standard deviation)

In considering a 3/1 adjustable-rate mortgage, the interest rate will be fixed for how many years?

3 years

While a variety of loan terms are available in a lender's mortgage menu, the most common loan term for level-payment mortgage is

30 years

While a variety of loan terms are available, the most common loan term on a fixed rate mortgage is:

30 years

A borrower takes out a 30 year mortgage loan for $100,000 with an interest rate of 6% plus 4 points. What is the blank if the loan is carried for all 30 years? a. 6.4% b. 7.8% c. 5.4% d. 4.8%

6.4%

Jenny needs to borrow $5,500 for four years. The loan will be repaid in one lump sum at the end of the loan term. Which one of the following interest rates is best for Jenny?

6.5 percent simple interest

municipal bond

Bonds issued by state and local governments are called _______ ______.

What type of management process function is being used when a manager motivates the workforce by rewarding them with additional vacation when standards are achieved?

Leading

Which one of the following provides compensation to a bondholder when a bond is not readily marketable at its full value?

Liquidity premium

Which loan in the above table is a Fixed Rate Mortgage

Loan 2 (No adjustment interval, Interest rate cap, or % Margin above index)

A credit card is a type of

Loan/borrowing.

You are evaluating a capital budgeting project that will cost $35,000 •Year 1 ==> $12,000 •Year 2 ==> $17,000 •Year 3 ==> $10,000 •Year 4 ==> $5,000 The required return is 15% and the critical acceptance level is 2.5 years. Calculate the Internal Rate of Return and determine whether or not the project should be accepted

The IRR is 11.32% and we should reject the project

You are evaluating a capital budgeting project that will cost $55,000 •Year 1 ==> $17,000 •Year 2 ==> $10,000 •Year 3 ==> $10,000 •Year 4 ==> $17,000 •Year 5 ==> $40,000 The required return is 12% and the critical acceptance level is 3.5 years. Calculate the Net Present Value and determine whether or not the project should be accepted based solely on NPV.

The NPV is $8,769.19 and we should accept the project

Under which of the following treaties are Canada, the United States, and Mexico gradually eliminating tariffs and all other trade barriers?

The North American Free Trade Agreement

An appraisal usually contains three approaches to valuation. What of the following is NOT one of the approaches?

The Ratio Approach

Treasury bonds, and treasury notes.

The US government borrows money by issuing:

Which country is the world's largest marketplace and most stable economy?

The United States

Which organization created the classification of countries based on per capita income?

The World Bank

You own a stock portfolio invested 30 percent in Stock Q, 14 percent in Stock R, 40 percent in Stock S, and 16 percent in Stock T. The betas for these four stocks are .99, 1.05, 1.45, and 1.90, respectively. What is the portfolio beta?

The beta of a portfolio is the sum of the weight of each asset times the beta of each asset. So, the beta of the portfolio is: βp = .30(.99) + .14(1.05) + .40(1.45) + .16(1.90) βp = 1.33

What condition must exist if a bond's coupon rate is to equal both the bond's current yield and its yield to maturity? Assume the market rate of interest for this bond is positive.

The bond must be priced at par.

Less

The bonds of a firm in financial distress may have a market value that is _____than the face value at maturity.

Who does NOT customarily attend real estate closing?

The buyer's and seller's immediate family

Differences between Roth IRAs and Traditional IRAs include ___.

Withdraws from Traditional IRAs are treated as ordinary income while withdraws from Roth IRAs are tax-free

If Kelsey...

Yes

The subject of an appraisal has only two bedrooms, but one of the comparables used in the...If the adjustment for a third bedroom is $5,000, the adjustment would be:

a $5,000 decrease to the comparable's selling price

Common sized balance sheet

a balance sheet in which a firm's assets and sources of debt and equity are expressed as a percentage of its total assets

Angel investor

a wealthy private investor who provides capital for a business start-up

What is the expected Rate of Return? State of Nature Probability Return Recession .20 -15% Normal .50 10% Boom .30 35%

k= (.20 * -15%) + (.50 * 10%) + (.30 * 35%) = 12.5%

Expected Return Formula

k=P1k1 + P2k2 + ..... + Pnkn k= the expected return on the stock Pi= the return probability of the ith possible outcome ki= the return under the ith outcome Pn= the probability of the nth possible outcome kn= the return under the nth outcome *Probabilities must sum to 1.0

E.R. for a Portfolio Equation

kp= W1k1 + W2k2 + .... Wnkn kp= the E.R. for the portfolio Wi= the weight (proportion) of stock 1 k1= expected return for stock 1 Wn= the weight (proportion) of stock n kn= E.R. for stock n

the Sarbanes-Oxley Act in 2002 was primary prompted by which one of the following from the 1990's?

Corporate accounting and financial fraud

John Walters is comparing the cost of credit to the cash price of an item. If John makes a down payment of $130 and pays $35 a month for 24 months, how much more will that amount be than the cash price of $713?

Cost of credit = Down payment + (Payment amount × Number of payments) - Cash price = $130 + ($35 × 24) − $713 = $257

Harrison Co. issued 16-year bonds one year ago at a coupon rate of 7.7 percent. The bonds make semiannual payments. If the YTM on these bonds is 5.4 percent, what is the current dollar price assuming a $1,000 par value?

Current bond price $ 1,234.40

Kendall is investing $3,333 today at 3 percent annual interest for three years. Which one of the following will increase the future value of that amount?

Increasing the interest rate

Which of the following is the BEST description of organizational stakeholders?

Individuals and groups that are directly affected by the practices of a company

A bank that provides overdraft protection charges 10 percent for each $100 (or portion of $100) borrowed when an overdraft occurs. a. What amount of interest would the customer pay for an overdraft of $260? (Assume the interest is for the full amount borrowed for a whole year.

Interest = Number of $100 increments × $100 × Interest rate = 3 × $100 × 0.100 = $30.00

If you have a $150,000 30-year 5% mortgage, how much of your first monthly payment of $805.50 would go toward principal?

Interest = Principal × interest rate × Time = $150,000 × 5% × 1 month/12 months = $625.00. Total payment - Interest = Principal = $805.50 - $625.00 = $180.50.

What are the interest cost and the total amount due on a six-month loan of $1,200 at 13 percent simple annual interest?

Interest cost (I) = P × r × T = $1,200 × 0.130 × (6 / 12) = $78 Total amount due = Interest + Principal = $78 + $1,200 = $1,278

You receive a credit card application from Shady Banks Savings and Loan offering an introductory rate of 2.2 percent per year, compounded monthly for the first six months, increasing thereafter to 17.1 percent compounded monthly. Assuming you transfer the $10,500 balance from your existing credit card and make no subsequent payments, how much interest will you owe at the end of the first year?

Interest owed $ 1,056.66 ± 0.1% Calculate FV of both and then subtract beginning balance from the ending balance to find interest accrued

What is the description of the risk positions of an ARM lender in comparison to that of a FRM lender?

Interest rate is lower, but default risk is higher.

Changes in interest rates affect bond prices. Which one of the following compensates bond investors for this risk?

Interest rate risk premium

Which of the following descriptions most accurately reflects the risk position of an ARM lender in comparison to that of a Fixed Rate Mortgage lender? Interest Risk Default Risk Lower Higher Higher Lower Lower Lower Higher Higher

Interest risk: Default Risk: Lower Higher

Travis borrowed $10,000 four years ago at an annual interest rate of 7 percent. The loan term is six years. Since he borrowed the money, Travis has been making annual payments of $700 to the bank. Which type of loan does he have?

Interest-only

Which one of the following is an example of systematic risk?

Investors panic causing security prices around the globe to fall precipitously.

While evaluating alternatives, all of the following are appropriate questions you could ask before making a major purchase except

Is it possible to delay the purchase or to do without the item? Should I pay for the item with cash or buy it on credit? Which brands should I consider? How do the price, quality, and service compare at different stores? ALL OF ABOVE

What do ethical norms that ensure an action is consistent with what's fair entail?

Justice

Blue Water bonds have a face value of $1,000, a coupon rate of 6.5 percent, semiannual interest payments, and mature in 11.5 years. What is the current price of these bonds if the yield to maturity is 6.36 percent?

$1,011.30

A borrower takes out a 30-year adjustable rate mortgage loan for $200,000 with monthly payments. The first two years of the loan have a "teaser rate of 4%, after that the rate can reset with a 2% annual rate cap. On the reset date, the composite rate is 5%. What would the year 3 monthly payment be? (A) $955 (B) $1,067 (C) $1,071 (D) $1,186 (E) Because of the rate cap, the payment would not change.

$1,067

You are buying a bond at a clean price of $1,140. The bond has a face value of $1,000, a coupon rate of 3.8 percent, and pays interest semiannually. The next coupon payment is one month from now. What is the dirty price of this bond?

$1,155.83

A property produces a first year NOI of $100,000 which is expected to grow by 2% per year. If the property is expected to be sold in year 10, what is the expected sale price based on a terminal capitalization rate applied to the eleventh year NOI?

$1,283,152

You are offered a 20-year mortgage at a fixed rate of 6%. If you choose to make...instead of monthly, how much will interest will you save?

$10,933

You are considering buying a foreclosed home as an investment, and expect to hold it for one year before selling it. You require a 25% rate of return on your investment. You are borrowing $110,000 as an interest only loan to make the purchase. Your acquisition costs will be $18,500, your holding period costs will be $73,000 and your expected sales costs will be $7,500. What is the amount of profit you must make to earn 25%? a. 110,000 b. 128,984 c. 105,667 d. 98,467

$105,667

Kristina started setting aside funds three years ago to save for a down payment on a house. She has saved $900 each quarter and earned an average rate of return of 4.8 percent. How much money does she currently have saved?

$11,542.10

A loan was made 10 years ago for $140,000 at 10.5% for a 30 year term. Rates are currently 9.25%. What is the market value of the loan? (A) $128,271 (B) $147,600 (C) $139,828 (D) $151,395

$139,828

Choose the correct statement regarding standard deviation

- If the standard deviation is zero, the actual return will always equal the expected return - All else equal, the higher the standard deviation the riskier the investment is as a stand-alone investment

Federal tax income policy has generally thought to: (A) Discourage homeownership (B) Encourage renting (C) Increase interest rates (D) Encourage homeownership

(D) Encourage home ownership

Capital Budgeting Process

*The most important factor in maximizing shareholder wealth. -Generating Ideas; gathering information and making cash flow estimates; make decision; evaluate/review

Which of the following would NOT be a legitimate investment to include in an IRA?

- Treasury Bonds - Mutual funds - Shares of common stock - Certificate of Deposit from a bank

A 401(k) Plan

- allows investments to compound tax-free while the money is in the 401(k) plan - generates an income stream during retirement that is based on the person's contributions and rate of return on those contributions.

Gorilla Movers has sales of $645,560, cost of goods sold of $425,890, depreciation of $32,450, and interest expense of $12,500. The tax rate is 30 percent. What is the times interest earned ratio?

0 14.98**** 0 12.75 0 11.63 0 6.25 0 2.75 Times interest earned ratio = ($645,560 - 425,890 - 32,450) / $12,500 = 14.98

A firm has total assets of $638,727, current assets of $203,015, current liabilities of $122,008, and total debt of $348,092. What is the debt-equity ratio?

0 1.03 0 1.20*** 0 1.31 0 1.43 O .87 Debt-equity ratio = $348,092 / ($638,727 - 348,092) = 1.20

Healthy Foods has a book value per share of $32.68, earnings per share of $3.09, and a priceearnings ratio of 16.8. What is the market-to-book ratio?

0 1.08 0 1.59*** 0 1.99 0 2.47 0 2.16 Market-to-book ratio = ($3.09 x 16.8)/$32.68 = 1.59

Of these choices, a risk-adverse investor who prefers to minimize interest rate risk is most apt to invest in:

2-year, 7 percent coupon bonds.

The Outpost, a sole proprietorship currently sells short leather jackets for $369 each. The firm is considering selling long coats also. The long coats would sell for $719 each and the company expects to sell 820 a year. If the company decides to carry the long coat, management feels that the annual sales of the short jacket will decline from 1,120 to 1,040 units. Variable costs on the jacket are $228 and $435 on the long coat. The fixed costs for this project are $23,100, depreciation is $10,400 a year, and the tax rate is 34 percent. What is the projected operating cash flow for this project?

-..> O $134,546 O $131,264 O $112,212 O $131,062 O $128,749 Operating cash flow = ([820 x ($719 — 435)] + [(1,040 - 1,120) x ($369 - 228)] - $23,100) x (1 - ($10,400 x .34) = $134,546

Maren's House of Pancakes has sales of $635,400, total equity of $268,000, and a debt-equity ratio of .6. What is the capital intensity ratio?

-..> O .67 O .59 O .72 O .89 0 1.67 Capital intensity ratio = [(1 + .6) x $268,000] / $635,400 = .67

One year ago, Alpha Supply issued 15-year bonds at par. The bonds have a coupon rate of 6.5 percent, paid semiannually, and a face value of $1,000. Today, the market yield on these bonds is 7.2 percent. What is the percentage change in the bond price over the past year?

-6.11 percent; PV = $938.89 Percentage change in price = ($938.89-1,000)/$1,000 = -.0611, or -6.11 percent

Beta

-A tool to measure how sensitive a stock is to the overall market. -Measures the degree of the market (non-diversifiable) risk -Tell us how a particular stock moves in relation to the rest of the stock market as a whole.

Important Implications of the CAPM/SML

-According to the SML, high beta stocks should, on average, earn higher returns than low beta stocks. -According to the SML, the only factor that should cause consistent differences in returns across stocks is beta. -When interest rates rise, required returns should increase (all else equal) cause stock prices to decline. -When investors become more risk-averse, the risk premium (km-kRF) should increase which will increase required returns and (all else equal) cause stock prices to decline.

Summary

-Average (mean) retirement savings for families in the 56-61 age bracket is $163,577 -Median is $17,000

Expected Return

-Based on the probability distribution of returns. -The probability of a specific state of nature occurring times the return under that state of nature summed across all possible states of nature.

How to interpret Beta:

-Betas > 1.0 implies higher than average risk -Betas = 1.0 implies average risk -Betas < 1.0 implies less than average risk -Most Betas range btw .35 and 1.8

Roth IRA

-Each individual can contribute up to $5500 per person per year into a Roth IRA. -Not tax break for your contribution -Contributions to a Roth IRA are NOT tax-deductible in the year of the contribution. -Eligible withdraws in retirement are tax-free income -Higher income restrictions -All interest, capital gains, and dividend income are allowed to compound tax-free during the investment period. -Withdrawals taken after age 59.5 (and minimum 5 years) and NOT taxed. This is a benefit.

Traditional IRA

-Each individual can contribute up to $5500 per person per year into an IRA. -Contributions to IRA are tax-deductible in the contribution year. -No taxes on investment income while in IRA -Eligible withdraws in retirement taxed as ordinary income. -Lower income restraints -Any withdrawals taken prior to age 59.5 are subject to a 10% tax penalty. -Withdrawals must be initiated by the time your 70.5

Adding a Security to a Well-Diversified Portfolio

-If your portfolio is well diversified, adding a single security is not very relevant because its one of many. What matters is how the stock movers with the overall market. Use BETA.

Payback Period

-Measures the amount of time it would take to earn back the initial investment in the project. Management then decides how long they are willing to wait to recover their investment and compares the calculated PP to the critical acceptance level. -Decision rule for independent projects is to accept all projects that have a PP < the critical acceptance level. -Mutually Exclusive projects, the project with the lowest PP would be chosen. Ex. PP period = 2.2 years. Critical acceptance level = 2. Reject! -If independt: accept all projects where PP < T -If M.E.: accept project with lowest PP < T

NPV

-Measures the value added by investing in the project. NPV is equal to the present value of all cash flows less the initial investment. -Decision rule for independent projects is to accept all projects with a +NPV. For M.E. projects, accept the project with the highest NPV. -The decision rule for NPV will always provide the correct decision. -When evaluating projects always use NPV as the decision maker. Even if PP and IRR conflict with your NPV analysis, go with the project with the highest NPV.

Mutual Fund Asset by Category

-Money market = 17% -Bond = 22% -Hybrid= 8% -Equity= 52%

Security Market Line (SML)

-States that the required RROR for a stock is dependent on the beta of that stock. kA= kRF + BA(km - kRF) kA= required return for Stock A kRF= the risk-free rate of interest BA= beta for stock A km= expected return on the market

2 situations when PP can help

-When the distance cash flows are highly uncertain. -When our firm is facing significant financial problems

Empirical Findings of the SML

-While market returns play a major role in explaining returns of individual stocks, Beta doesnt do a very good job of explaining future returns. -Small firms seem to earn higher returns that can be explained by beta. -Firms with a low market-to-book ratio tend to earn higher returns than can be explained by beta. -Firms that have been top performers in the past 6-12 months tend to earn higher returns in the following 6-12 months that can be explained by beta.

The portfolio weight of an asset?

-is the total investment in that asset divided by the total portfolio value. First, we will find the portfolio value.

Gilmore, Inc., just paid a dividend of $2.90 per share on its stock. The dividends are expected to grow at a constant rate of 4.75 percent per year, indefinitely. Assume investors require a return of 9 percent on this stock. 1.) What is the current price? 2.) What will the price be in six years and in thirteen years?

1.) Pt = Dt × (1 + g) / (R - g) P0 = $2.90(1.0475) / (.09 - .0475) P0 = $71.48 2.) P6 = D6(1 + g) / (R - g) P6 = D0(1 + g)7 / (R - g) P6 = $2.90(1.0475)7 / (.09 - .0475) P6 = $94.43 We can do the same thing to find the dividend in Year 14, which gives us the price in Year 13, so: P13 = D13(1 + g) / (R - g) P13 = D0(1 + g)14 / (R - g) P13 = $2.90(1.0475)14 / (.09 - .0475) P13 = $130.67 P6 = P0(1 + g)6 P6 = $71.48(1 + .0475)6 P6 = $94.43 P6 = P0(1 + g)6 P6 = $71.48(1 + .0475)6 P6 = $94.43 And the stock price in Year 13 will be: P13 = P0(1 + g)13 P13 = $71.48(1 + .0475)13 P13 = $130.67

The common stock of Jensen Shipping has an expected return of 14.7 percent. The return on the market is 10.8 percent and the risk-free rate of return is 3.8 percent. What is the beta of this stock?

1.56

You are evaluating a capital budgeting project that will cost $25,000 •Year 1 ==> $12,000 •Year 2 ==> $20,000 •Year 3 ==> $9,000 The required return is 13% and the critical acceptance level is 1.9 years. Calculate the Payback Period and determine whether or not the project should be accepted based solely on the Payback Period.

1.65 years and accept the project To calculate the Payback Period, we count how long it takes us to recover our initial investment. In this problem, we spend $25,000 initially and get $13,000 back in year one. This gets our count to one. Then, we look at year two and realize that we will have MORE than paid back the initial investment by the end of year two. Therefore, our count will not reach two and our PP will be between one and two years. After year one, we have $13,000 yet to recover and we will get $20,000 in year 2, so we take the 1 year plus 13,000/20,000 and get a Payback Period of 1.65 years. Since this is LESS THAN the 1.9 year critical acceptance level we ACCEPT the project.

You own a portfolio equally invested in a riskfree asset and two stocks. If one of the stocks has a beta of 1.15 and the total portfolio is equally as risky as the market, what must the beta be for the other stock in your portfolio?

1.85

What are the arithmetic and geometric average returns for a stock with annual returns of 21 percent, 9 percent, -2 percent, and 13 percent? List the arithmetic answer first.

10.25 percent; 9.93 percent

Delmont Movers has a profit margin of 7.1 percent and net income of $63,700. What is the common-size percentage for the cost of goods sold if that expense amounted to $522,600 for the year?

12.19 percent 23.50 percent 0 53.25 percent 61.06 percent 0 58.25 percent*** COGS common-size percentage = $522,600 / ($63,700 / .071) = .5825, or 58.25 percent

Consider the following probability distribution •Probability Return • 0.10 -20% • 0.20 5% • 0.40 12% • 0.30 30% Calculate the expected return for this security.

12.80% expected return = 0.1(-20%) + 0.2(5%) + 0.4(12%) + 0.3(30%) = -2% + 1% + 4.8% + 9% = 12.8%

Suppose a stock had an initial price of $57 per share, paid a dividend of $1.7 per share during the year, and had an ending share price of $63. Compute the percentage total return.

13.51

An amortized, 3-year loan has annual payments and an effective annual rate of 14.56 percent. What is the APR?

14.56 percent; The APR equals the EAR when interest rate compounding is annual.

You own a portfolio that has $2,250 invested in Stock A and $3,500 invested in Stock B. If the expected returns on these stocks are 12 percent and 17 percent, respectively, what is the expected return on the portfolio?(Do not round your intermediate calculations.)

15.04%

Consider the following probability distribution •Probability Return • 0.25 -20% • 0.50 10% • 0.25 36% Calculate the standard deviation for this security.

19.82% Exp. Return = 0.25(-20%) + 0.5(10%) + 0.25(36%) = -5% + 5% + 9% = 9%. Once we have the Exp. Return, we can estimate the St. Deviation. St. Dev = [0.25(-20 - 9)^2 + 0.5(10 - 9)^2 + 0.25(36 - 9)^2]^0.5 = [210.25 + 0.5 + 182.25]^0.5 = [393]^0.5 = 19.82%

A typical RESPA closing statement has which of the following characteristics?

2 columns - Summary of borrower's and seller's transaction

401k Plans

A retirement savings plan offered through an employer that allows employees to put away a portion of their paycheck each period on a pre-tax basis into an investment plan. The money in the 401k compounds tax-free during the employee's working life and withdraws are taxed as ordinary income. Max contribution for an employee is $18,000 ($24,000 for 50+ years of age) per year.

Choose the correct statement regarding ETFs

An ETF is a security that is structured like a mutual fund (each share represents a portfolio of securities), but trades like a stock (bought sold through a brokerage account on stock exchanges)

Will wants to open an account with $500 that will allow him easy access to his funds (by checks, ATMs, or debit card usage) from many locations. He also wants to earn at least a low interest rate and have federal deposit insurance. What kind of account should he open?

An interest-earning checking account

Load Charge

An upfront fee that is taken from your investment. Usually used to help pay for the sales costs associated with marketing mutual funds.

Maybepay Life Insurance Co. is selling a perpetual annuity contract that pays $2,900 monthly. The contract currently sells for $327,000. What is the effective annual return?

And using the equation to find the EAR, we find: EAR = [1 + (APR / m)]m - 1 EAR = [1 + .0089]12 - 1 EAR = .1118, or 11.18%

A payday loan company charges 4.8 percent interest for a two-week period. What would be the annual interest rate from that company? Assume an even 52 weeks per year

Annual interest rate = Interest rate per period × Number of periods per year = 0.048 × (52 / 2) = 1.248, or 124.8%

What would be the annual percentage yield for a savings account that earned $71 in interest on $1,000 over the past 365 days?

Annual percentage yield = Annual interest / Principal = $71 / $1,000 = 0.071, or 7.1%

Travis is buying a car and will finance it with a loan that requires monthly payments of $265 for the next four years. His car payments can be described by which one of the following terms?

Annuity

Most people would tend to agree that technology stocks were highly overvalued in the late 1990's. This time period is best described as a technology:

Bubble

-The maximum reward for owning debt is fixed. -Equity represents an ownership interest.

As a general rule, which of the following are true of debt and equity?

Which of the following correctly describes the relationship between depreciation, income, taxes, and investment cash flows?

As depreciation expense increases, net income and taxes will decrease, while investment cash flows will increase.

Which of the following is correct?

Bank customers typically are provided with online access to view and print checks that have been paid.

A preferred stock offers a rate of return of 5.45 percent and sells for $78.20? What is the annual dividend amount?

C = .0545 x $78.20 = $4.26

The manager of Gloria's Boutique has approved Carla's application for 24 months of credit with maximum monthly payments of $70.If the APR is 14.2 percent, what is the maximum initial purchase that Carla can buy on credit?

C. $1,455.08

Cromwell is acquiring some land for $1,200,000 in exchange for semiannual payments of $75,000 at an interest rate of 6.35 percent. How many years will it take Cromwell to pay for this purchase?

C. 11.35 years

Which one of the following is a correct ranking of securities based on their volatility over the period of 1926-2010? Rank from highest to lowest. A. Large company stocks, U.S. Treasury bills, long-term government bonds. B. Small company stocks, long-term corporate bonds, large company stocks. C. Small company stocks, long-term corporate bonds, intermediate-term government bonds. D. Large company stocks, small company stocks, long-term government bonds. E. Intermediate-term government bonds, long-term corporate bonds, U.S. Treasury bills.

C. Small company stocks, long-term corporate bonds, intermediate-term government bonds.

Choosing btw 2 (or more) Well-Diversified Portfolios

Can use either SD or BETA bc the firm-specific risk is already diversified. Thus, whichever portfolio has the higher SD should also have the higher beta.

A loan officer is examining whether or not to offer you a loan today. Specifically, she is examining your income and debts. Which of the five Cs of credit is the loan officer reviewing?

Capacity

Craig wants to get his old car repaired, but he doesn't want to spend much money. He probably will NOT want to go to the most expensive repair service, which is from a(n)

Car dealer.

Crossover (Multiple IRR) Problem

Cash flows go from - in one period to + in the next or vice versa. IRR cannot be used bc the projects are of different size and the timing of cash flows is vastly different. This is referred to as the Reinvestment Rate Problem.

Which of the following is NOT correct?

Check 21 increases the fees banks can charge for checking accounts

Which of the following should be the first step in assessing ethical behavior in a certain situation?

Collect facts related to the situation.

Property rights can be divided into to classes; real and personal. What is an example of real property?

Commercial Building

The idea that individuals would contribute according to their abilities and receive benefits according to their needs supports which type of economy?

Communist based economy

You want to buy a new sports coupe for $74,500, and the finance office at the dealership has quoted you a loan with an APR of 6.9 percent for 36 months to buy the car. What is the effective annual rate on this loan?

Effective annual rate 7.12 ± 1%

PK Software has 7.8 percent coupon bonds on the market with 25 years to maturity. The bonds make semiannual payments and currently sell for 108.75 percent of par. What is the effective annual yield?

Effective annual yield 7.17 %

Who is the person who accepts the risks and opportunities inherent in a new business opportunity?

Entrepreneurs

Which one of the following can be classified as an annuity but not as a perpetuity?

Equal annual payments for life

Which of the following is an account used to pay property taxes and homeowner's insurance?

Escrow

Interpreting Expected Return and SD

Expected Return gives us an idea oh how much we will make on the investment. It is NOT how much we WILL make, but how much we would make ON AVERAGE if we could repeat the holding period an infinite # of times.

What does an expense stop do in a lease?

Expenses above the stop are paid by the tenant

Bruer, Inc., is expected to maintain a constant 5.8 percent growth rate in its dividends, indefinitely. If the company has a dividend yield of 4.3 percent, what is the required return on the company's stock?

Explanation: The required return of a stock is made up of two parts: The dividend yield and the capital gains yield. So, the required return of this stock is: R = Dividend yield + Capital gains yield R = .043 + .058 R = .1010 or 10.10%

Bowman Corp. pays a constant $13 dividend on its stock. The company will maintain this dividend for the next eight years and will then cease paying dividends forever. If the required return on this stock is 9 percent, what is the current share price?

Explanation: The price of any financial instrument is the present value of the future cash flows. The future dividends of this stock are an annuity for eight years, so the price of the stock is the present value of an annuity, which will be: P0 = $13.00(PVIFA9%,8) P0 = $71.95 Calculator solution: You can use the TVM function as follows: N = 8 I = 9 PMT = 13 FV = 0 PV = to figure out for price You can also use the NPV function as follows: Npv= (9, 0, {13}, {8}) = to get the price

What are products that are created domestically and transported for sale abroad?

Exports

A borrower finds that the incremental cost of borrowing an extra $10,000 is 14%. If a loan is obtained at 15% so the borrower would be better off by borrowing with the loan rate mortgage.

FALSE

A gross income multiplier can be calculated by dividing the gross income by the sales price

FALSE

A gross lease is where tenants pay all expenses

FALSE

Business ethics refers to ethical or unethical behaviors by employees in the context of their personal lives.

FALSE

Most businesses in the United States are large businesses with more than 500 employees.

FALSE

Physical resources include the data and other information used by businesses. T/F

FALSE

T/F: A property purchased for $350,000. Based on an annual growth rate of 3%, the resale value at the end of year 10, would be $456,671.

FALSE

The Small Business Administration is the government agency charged with owning small businesses.

FALSE

The customs, values, and demographic characteristics of the society in which an organization functions are the principal elements of the political-legal environment. T/F?

FALSE

The first step in the control process is to compare actual performance to standards

FALSE

McClary Tires plans to save $20,000, $25,000, $27,500, and $30,000 at the end of each year for Years 1 to 4, respectively. If it earns 3.3 percent on its savings, how much will the firm have saved at the end of Year 4?

FV = ($20,000 x1.0333) + ($25,000 x1.0332) + ($27,500 x1.0331) + $30,000 = $107,130.78

A service contract for a video projection system costs $245 a year. You expect to use the system for three years. Instead of buying the service contract, what would be the future value of these annual amounts after three years if you earn 5 percent on your savings?

FV = Annual cost × Future value annuity factor = $245 × 3.153 = $772.49

Joshua borrowed $600 for one year and paid $30 in interest. The bank charged him a service charge of $9. If Joshua paid the $600 in 12 equal monthly payments, what is the APR?

Finance charge = Interest + Service charge = $30 +$9 = $39 APR = (2 × n × I) / [P(N + 1)] = (2 × 12 × $39) / [$600(12 + 1)] = 0.120, or 12.0%

Joshua borrowed $1,000 for one year and paid $50 in interest. The bank charged him a service charge of $7. What is the finance charge on this loan?

Finance charge = Interest + Service charge = $50 + $7 = $57

CFO and controller are titles associated with what type of manager?

Financial

Relationships determined from a firm's financial information and used for comparison purposes are known as:

Financial Ratios

For an investor that plans to make regular contributions to a mutual fund over a long period of time (say $3000 per year for 30 years), the load charge is more important than the expense ratio in impacting net wealth at the end of the time period.

False

You are considering the purchase of a stock with a beta of 1.6. The current risk-free rate of interest is 5% and the expected return on the market is 11%. You have estimated that the expected return for this stock is 13%. Based on this information, you should purchase the stock.

False

Factors such as international trade agreements, economic conditions, and political unrest will have the greatest impact on what type of business environment?

Global business environment

What term BEST describes a person that owns a property and is conveying title to the property to another person?

Grantor

You have $12,000 to invest in a stock portfolio. Your choices are Stock X with an expected return of 16 percent and Stock Y with an expected return of 9 percent. Assume your goal is to create a portfolio with an expected return of 14.15 percent. How much money will you invest in Stock X and Stock Y?

Here, we are given the expected return of the portfolio and the expected return of each asset in the portfolio, and are asked to find the weight of each asset. We can use the equation for the expected return of a portfolio to solve this problem. Since the total weight of a portfolio must equal 1 (100%), the weight of Stock Y must be one minus the weight of Stock X. Mathematically speaking, this means: E(Rp) = .1415 = .16wX + .09(1 - wX) We can now solve this equation for the weight of Stock X as: .1415 = .16wX + .09 - .09wX .0515 = .07wX wX = .7357 So, the dollar amount invested in Stock X is the weight of Stock X times the total portfolio value, or: Investment in X = .7357($12,000) Investment in X = $8,828.57 And the dollar amount invested in Stock Y is: Investment in Y = (1 - .7357)($12,000) Investment in Y = $3,171.43

A stock has an expected return of 11.2 percent, its beta is .87, and the risk-free rate is 5.6 percent. What must the expected return on the market be?

Here, we need to find the expected return of the market, using the CAPM. Substituting the values given, and solving for the expected return of the market, we find: E(Ri) = Rf + [E(RM) - Rf] × βi .112 = .056 + [E(RM) - .056](.87) E(RM) = .1204, or 12.04%

A stock has an expected return of 13.7 percent and a beta of 1.18, and the expected return on the market is 12.7 percent. What must the risk-free rate be?

Here, we need to find the risk-free rate, using the CAPM. Substituting the values given, and solving for the risk-free rate, we find: E(Ri) = Rf + [E(RM) - Rf] × βi .137 = Rf + (.127 - Rf)(1.18) .137 = Rf + .14986 - 1.18Rf Rf = .0714, or 7.14%

What caused manufacturing jobs to move from Mexico to China in the late 1990s?

Increases in Mexico's cost of living and wages

Which of the following best explains the difference between a market economy and a planned economy?

In a planned economy, consumers have little choice in where they work or what they purchase or pay.

Capitalization rates will differ from yield rates when the income is expected to ________ over time.

Increase and Decrease. Capitalization rates equal yield rates only when income doesn't change.

Which of the following is correct about the lemon law?

It applies to vehicles that have been serviced four times to correct the same problem in the first 12,000 miles.

What are the two main drawbacks of sensitivity analysis?

It does not consider interaction among variables It may increase the false sense of security among managers if all pessimistic estimates of NPV are positive.

In the context of capital budgeting, what does sensitivity analysis do?

It examines how sensitive a particular NPV calculation is to changes in underlying assumptions.

What is true concerning capitalization rates?

It expresses relationships between income and property value at a specific point in time

What are the two main benefits of performing sensitivity analysis?

It identifies the variable that has the most effect on NPV. It reduces a false sense of security by giving a range of values for NPV instead of a single value.

What is an important drawback of traditional NPV analysis?

It ignores managerial options in investment decisions.

Why is it difficult to compare relative job growth for different-sized businesses?

It is hard to determine the cutoff point at which a small business becomes a large business.

Which of the following are reasons why NPV is considered a superior capital budgeting technique?

It properly chooses among mutually exclusive projects It considers time value of money. It considers all the cash flows. It considers the riskiness of the project.

Which is TRUE regarding the incremental Cost of Borrowing?

It should be compared to the cost of obtaining a second mortgage

E.R. for a Portfolio

Its the weighted average of each stock held in the portfolio.

Janie has a joint account with her mother with a balance of $626,000. Based on $250,000 of Federal Deposit Insurance Corporation coverage, what amount of Janie's savings would not be covered by deposit insurance?

Janie's portion of joint account = 0.50 × $626,000 = $313,000 Uninsured portion of Janie's account = Janie's portion of joint account - FDIC coverage amount = $313,000 - $250,000 = $63,000

Small businesses impact key aspects of the U.S. economy including

Job creation and innovation

What document conveys title to a property at the time the purchaser completes the performance of the obligation called for in the document? It is often used in seller financing.

Land contract

Standards that help to shape behaviors towards employees, the organization, and other economic agents are

Managerial ethics

Which business constituents analyze their competitive environments and plan, organize, direct, and control the operations of their organizations?

Managers

Which one of the following is represented by the slope of the security market line?

Market Risk Premium

In which type of economy do the individual producer and consumer control production through supply and demand?

Market economy

When making decisions about production and allocation, which type of system depends on individual producers and consumers to create a combination of supply and demand?

Market economy

In a small business, who determines prices for wholesalers and customers?

Market forces

What is the price at which the quantity of goods demanded and the quantity of goods supplied are equal?

Market price

Last year, T-bills returned 2 percent while your investment in large-company stocks earned an average of 5 percent. Which one of the following terms refers to the difference between these two rates?

Risk Premium

What is risk

Risk refers to the possibility of an unfavorable event occurring.

Rollover IRA

Rollover IRA's are designed to allow individuals to move their pension plan into a taxsheltered account when they switch jobs, essentially turning the old pension plan into a traditional IRA.

What is the difference between scenario analysis and sensitivity analysis?

Scenario analysis considers a combination of factors for each scenario while sensitivity analysis focuses on only one variable at a time.

The text discusses several types of bonds. Which bond pays interest for 30 years that is exempt from state and local taxes?

Series EE

Which of the following industry groups is the fastest growing segment of small business enterprise?

Services

Jessica invested $2,000 today in an investment that pays 6.5 percent annual interest. Which one of the following statements is correct, assuming all interest is reinvested?

She could have the same future value and invest less than $2,000 initially if she could earn more than 6.5 percent interest.

capital budgeting includes the evaluation of

Size, timing, and risk of future cash flows

Which entity is defined as independent and has little influence on its market?

Small business

Maybepay Life Insurance Co. is selling a perpetual annuity contract that pays $2,900 monthly. The contract currently sells for $327,000. What is the APR?

The interest rate is .89 percent per month. To find the APR, we multiply this rate by the number of months in a year, so: APR = (12).89% APR = 10.64%

What is the willingness and ability of producers to offer a good for sale referred to as?

Supply

The __________ tells us that the expected return on a risky assets depends only on that asset's non-diversifiable risk.

Systematic Risk Principle

Which one of the following types of bonds should an investor purchase if he or she is primarily concerned about ensuring that bond ownership will increase his or her purchasing power?

TIPS

A conflict of interest occurs when an activity may benefit the individual to the detriment of his or her employer.

TRUE

An import is a product made or grown abroad but sold domestically

TRUE

An organizations external environment consists of everything that might affect it.T/F

TRUE

An overall capitalization rate can be calculated by dividing the net operating income by the property value

TRUE

Ethics are beliefs about what is right and wrong.

TRUE

If interest rates decrease, the market value of a loan previously make will increase

TRUE

Small businesses produce more patents per employee than large patenting firms.

TRUE

TRUE/FALSE: A clause which specifies that the mortgagor will pay all property taxes and other charges assessed against the property, eve if these charges have priority over the mortgage is typically included in mortgage.

TRUE

The World Trade Organization (WTO) was created to encourage international trade

TRUE

The pursuit of profits is how a business differs from organizations such as public universities, public hospitals, and government agencies. T/F

TRUE

When a sales manager compares actual sales to the quarterly sales quota, he or she is performing the controlling function of management.

TRUE

You own a stock portfolio invested 16 percent in Stock Q, 24 percent in Stock R, 36 percent in Stock S, and 24 percent in Stock T. The betas for these four stocks are .94, 1.00, 1.40, and 1.85, respectively. What is the portfolio beta?

The beta of a portfolio is the sum of the weight of each asset times the beta of each asset. So, the beta of the portfolio is: βp = .16(.94) + .24(1.00) + .36(1.40) + .24(1.85) βp = 1.34

longer-term

The interest rate risk premium is the additional compensation demanded by investors for holding ____ bonds.

With a 28 percent marginal tax rate, would a tax-free yield of 5.4 percent or a taxable yield of 8.5 percent give you a better return on your savings?

Tax-free yield: Aftertax yield = Pretax yield = 8.5% Taxable yield: Aftertax yield = Taxable yield × (1 - Tax rate) = 0.085 × (1 - 0.28) = 0.0612, or 6.12%

Which of the following statements regarding the relationship between book value, sales price, and taxes are true when a firm sells a fixed asset?

Taxes are based on the difference between the book value and the sales price; Book value represents the purchase price minus the accumulated depreciation; There will be a tax savings if the book value exceeds the sales price.

To encourage borrowers to accept ARMs rather than level-payment mortgages, mortgages originators generally offer an initial, short-term introductory rate that is less than previously-offered mortgage rate

Teaser Rate

Which type of business environment can reduce or replace the need for traditional working methods, physical equipment, and other platforms needed to conduct business?

Technological environment

Kenny, Inc., is looking at setting up a new manufacturing plant in South Park. The company bought some land six years ago for $7 million in anticipation of using it as a warehouse and distribution site, but the company has since decided to rent facilities elsewhere. The land would net $9.8 million if it were sold today. The company now wants to build its new manufacturing plant on this land; the plant will cost $21 million to build, and the site requires $850,000 worth of grading before it is suitable for construction. What is the proper cash flow amount to use as the initial investment in fixed assets when evaluating this project?

The $7 million acquisition cost of the land six years ago is a sunk cost. The $9.8 million current aftertax value of the land is an opportunity cost if the land is used rather than sold off. The $21 million cash outlay and $850,000 grading expenses are the initial fixed asset investments needed to get the project going. Therefore, the proper year zero cash flow to use in evaluating this project is Cash flow = $9,800,000 + 21,000,000 + 850,000 Cash flow = $31,650,000

A stock has a beta of 1.2, the expected return on the market is 11.4 percent, and the risk-free rate is 4.75 percent. What must the expected return on this stock be?

The CAPM states the relationship between the risk of an asset and its expected return. The CAPM is: E(Ri) = Rf + [E(RM) - Rf] × βi Substituting the values we are given, we find: E(Ri) = .0475 + (.1140 - .0475)(1.20) E(Ri) = .1273, or 12.73%

Which one of the following statements is correct?

The EAR, rather than the APR, should be used to compare both investment and loan options.

A stock has had returns of −19.9 percent, 29.9 percent, 34.8 percent, −11 percent, 35.7 percent, and 27.9 percent over the last six years. What are the arithmetic and geometric returns for the stock?

The arithmetic average return is the sum of the known returns divided by the number of returns, so: Arithmetic average return = (-.199 + .299 + .348 - .110 + .357 + .279) / 6 Arithmetic average return = .1623, or 16.23% Using the equation for the geometric return, we find: Geometric average return = [(1 + R1) × (1 + R2) × ... × (1 + RT)]1/T - 1 Geometric average return = [(1 - .199)(1 + .299)(1 + .348)(1 - .110)(1 + .357)(1 + .279)](1/6) - 1 Geometric average return = .1375, or 13.75% Remember, the geometric average return will always be less than the arithmetic average return if the returns have any variation.

You own a portfolio equally invested in a risk-free asset and two stocks. One of the stocks has a beta of 1.14 and the total portfolio is equally as risky as the market. What must the beta be for the other stock in your portfolio?

The beta of a portfolio is the sum of the weight of each asset times the beta of each asset. If the portfolio is as risky as the market, it must have the same beta as the market. Since the beta of the market is one, we know the beta of our portfolio is one. We also need to remember that the beta of the risk-free asset is zero. It has to be zero since the asset has no risk. Setting up the equation for the beta of our portfolio, we get: βp = 1.0 = 1/3(0) + 1/3(1.14) + 1/3(βX) Solving for the beta of Stock X, we get: βX = 1.86

You own a portfolio that is 31 percent invested in Stock X, 46 percent in Stock Y, and 23 percent in Stock Z. The expected returns on these three stocks are 11 percent, 14 percent, and 16 percent, respectively. What is the expected return on the portfolio?

The expected return of a portfolio is the sum of the weight of each asset times the expected return of each asset. So, the expected return of the portfolio is: E(Rp) = .31(.11) + .46(.14) + .23(.16) E(Rp) = .1353, or 13.53%

You own a portfolio that has $2,900 invested in Stock A and $4,000 invested in Stock B. Assume the expected returns on these stocks are 8 percent and 14 percent, respectively. What is the expected return on the portfolio?

The expected return of a portfolio is the sum of the weight of each asset times the expected return of each asset. The total value of the portfolio is: Total value = $2,900 + 4,000 Total value = $6,900 So, the expected return of this portfolio is: E(Rp) = ($2,900 / $6,900)(.08) + ($4,000 / $6,900)(.14) E(Rp) = .1148, or 11.48%

You own a portfolio that has $3,200 invested in Stock A and $4,300 invested in Stock B. Assume the expected returns on these stocks are 12 percent and 18 percent, respectively. What is the expected return on the portfolio?

The expected return of a portfolio is the sum of the weight of each asset times the expected return of each asset. The total value of the portfolio is: Total value = $3,200 + 4,300 Total value = $7,500 So, the expected return of this portfolio is: E(Rp) = ($3,200 / $7,500)(.12) + ($4,300 / $7,500)(.18) E(Rp) = .1544, or 15.44%

Which of the following qualify as "managerial options"?

The option to wait The option to expand The option to abandon

Today, Charity wants to invest less than $3,000 with the goal of receiving $3,000 back some time in the future. Which one of the following statements is correct?

The period of time she has to wait decreases as the amount she invests increases.

What are the portfolio weights for a portfolio that has 175 shares of Stock A that sell for $92 per share and 150 shares of Stock B that sell for $114 per share?

The portfolio weight of an asset is the total investment in that asset divided by the total portfolio value. First, we will find the portfolio value, which is: Total value = 175($92) + 150($114) Total value = $33,200 The portfolio weight for each stock is: xA = 175($92) / $33,200 xA = .4849 or 48.49 xB = 150($114) / $33,200 xB = .5151 or 51.51

You are comparing three investments, all of which pay $100 a month and have an interest rate of 8 percent. One is ordinary annuity, one is an annuity due, and the third investment is a perpetuity. Which one of the following statements is correct given these three investment options?

The present value of the perpetuity has to be higher than the present value of either the ordinary annuity or the annuity due.

The market value of a loan is: (A) The loan balance times one minus the market rate (B) The loan balance times one minus the original rate (C) The future value of the remaining payments (D) The present value of the remaining payments

The present value of the remaining payments

When lenders charge discount points (prepaid interest) on a loan, what impact does this have on the lease yield?

The yield on the loan will increase

Sole proprietorship

a business owned by an individual

Reinvestment Rate Problem

This bias will be > for projects that are Frontloaded. Frontloaded refers to projects with higher cash flows early in the project life. This bias is > here bc the faulty reinvestment rate assumption has longer to impact our final answer. This bias is smaller for projects that are Backloaded. Bc of this difference in bias, Frontloaded projects are likely to have an artificially higher IRR than Backloaded projects causing us to make poor rankings. If we are evaluating M.E. projects with differnet timing, then we should be careful of the reinvestment rate problem and choose NPV as our decision tool.

In considering a 3/1 ARM, the interest rate will be fixed for how many years?

Three years

How does globalization of business benefit shareholders of an organization?

Through increased and improved profitability

Increases

When interest rates in the market fall, bond values will increase because the present value of the bond's remaining cash flows ____.

A stock has had the following year-end prices and dividends: Year Price Dividend 1 .)$ 64.73 — 2.) 71.60 $ .68 3 .)77.40 .73 4.)63.67 .79 5.) 73.91 .88 6.) 83.75 .95 What are the arithmetic and geometric returns for the stock?

To calculate the arithmetic and geometric average returns, we must first calculate the return for each year. The return for each year is: R1 = ($71.60 - 64.73 + .68) / $64.73 = .1166, or 11.66% R2 = ($77.40 - 71.60 + .73) / $71.60 = .0912, or 9.12% R3 = ($63.67 - 77.40 + .79) / $77.40 = -.1672, or -16.72% R4 = ($73.91 - 63.67 + .88) / $63.67 = .1747, or 17.47% R5 = ($83.75 - 73.91 + .95) / $73.91 = .1460, or 14.60% The arithmetic average return was: RA = (.1166 + .0912 - .1672 + .1747 + .1460) / 5 RA = .0723, or 7.23% And the geometric average return was: RG = [(1 + .1166)(1 + .0912)(1 - .1672)(1 + .1747)(1 + .1460)]^1/5 - 1 RG = .0644, or 6.44%

A stock has had the following year-end prices and dividends: Year Price Dividend 1 $ 64.88 — 2 71.75 $ .71 3 77.55 .76 4 63.82 .82 5 74.21 .91 6 85.25 .98 What are the arithmetic and geometric returns for the stock?

To calculate the arithmetic and geometric average returns, we must first calculate the return for each year. The return for each year is: R1 = ($71.75 - 64.88 + .71) / $64.88 = .1168, or 11.68% R2 = ($77.55 - 71.75 + .76) / $71.75 = .0914, or 9.14% R3 = ($63.82 - 77.55 + .82) / $77.55 = -.1665, or -16.65% R4 = ($74.21 - 63.82 + .91) / $63.82 = .1771, or 17.71% R5 = ($85.25 - 74.21 + .98) / $74.21 = .1620, or 16.20% The arithmetic average return was: RA = (.1168 + .0914 - .1665 + .1771 + .1620) / 5 RA = .0762, or 7.62% And the geometric average return was: RG = [(1 + .1168)(1 + .0914)(1 - .1665)(1 + .1771)(1 + .1620)]1/5 - 1 RG = .0680, or 6.80%

You purchased 250 shares of a particular stock at the beginning of the year at a price of $76.13. The stock paid a dividend of $1.35 per share, and the stock price at the end of the year was $82.64. What was your dollar return on this investment?

To calculate the dollar return, we multiply the number of shares owned by the change in price per share and the dividend per share received. The total dollar return is: Dollar return = 250($82.64 - 76.13 + 1.35) Dollar return = $1,965.00

You purchased 320 shares of a particular stock at the beginning of the year at a price of $76.83. The stock paid a dividend of $1.70 per share, and the stock price at the end of the year was $83.34. What was your dollar return on this investment?

To calculate the dollar return, we multiply the number of shares owned by the change in price per share and the dividend per share received. The total dollar return is: Dollar return = 320($83.34 - 76.83 + 1.70) Dollar return = $2,627.20

You've observed the following returns on Barnett Corporation's stock over the past five years: -24.9 percent, 13.6 percent, 30.2 percent, 2.3 percent, and 21.3 percent. What was the arithmetic average return on the stock over this five-year period? What was the variance of the returns over this period? What was the standard deviation of the returns over this period?

To find the average return, we sum all the returns and divide by the number of returns, so: Arithmetic average return = (-.249 + .136 + .302 + .023 + .213) / 5 Arithmetic average return = .0850, or 8.50% Using the equation to calculate variance, we find: Variance = 1/4[(-.249 - .0850)2 + (.136 - .0850)2 + (.302 - .0850)2 + (.023 - .0850)2 + (.213 - .0850)2] Variance = .045369 So, the standard deviation is: Standard deviation = .0453691/2 Standard deviation = .2130, or 21.30%

You purchased a zero-coupon bond one year ago for $278.33. The market interest rate is now 8 percent. Assume semiannual compounding periods. If the bond had 17 years to maturity when you originally purchased it, what was your total return for the past year?

To find the return on the zero-coupon bond, we first need to find the price of the bond today. We need to remember that the price for zero-coupon bonds is calculated with semiannual periods. Since one year has elapsed, the bond now has 16 years to maturity, so the price today is: P1 = $1,000 / 1.04032 P1 = $285.06 There are no intermediate cash flows on a zero-coupon bond, so the return is the capital gain, or: R = ($285.06 - 278.33) / $278.33 R = .0242, or 2.42%

Which level of management is responsible for the overall performance and effectiveness of the firm?

Top managers

Standard deviation measures which type of risk?

Total

Suppose a stock had an initial price of $72 per share, paid a dividend of $1.20 per share during the year, and had an ending share price of $61. Compute the percentage total return. What was the dividend yield and the capital gains yield?

Total Returns -13.61% Dividend Yield 1.67% Capital Gains Yield -15.28%

decrease

When interest rates in the market rise, we can expect the price of bonds to ____.

Upward sloping

When long-term rates are higher than short-term rates, which of the following shapes will the term structure of interest rates usually have?

As long as the correlation is < 1.0 (which it will be for any 2 stocks), the risk of the portfolio is < the weighted average risk of the two securities which make up the portfolio.

True

Including a municipal bond in an IRA would generally be a bad idea.

True

Mortgage interest and property taxes are deductible for federal income tax purposes for homeowners True or False

True

Potential investors in analyzing the profit potential for a distressed property, generally consider a financial framework including the acquisition blank holding period phase and the disposition phase True or False

True

The incremental cost of borrowing may also be referred to as the marginal cost of borrowing True or False

True

True

True or false: Long-term debt has maturities greater than one year.

true

True or false: The government sells Treasury notes and bonds to the public every month.

true

True or false: Zero coupon bond calculations use semiannual periods to be consistent with coupon bond calculations.

Which one of these is a perpetuity?

Trust income of $1,200 a year forever

Which one of the following categories of securities had the lowest average risk premium for the period 1926-2010?

U.S. Treasury Bills

Which one of the following categories of securities had the lowest average risk premium for the period 1926-2010?

U.S. Treasury bills

Limited liability Company (LLC)

a cross between a partnership and corporation under which the owners retain limited liability but the company is run and taxed like a partnership

Take Time Corporation will pay a dividend of $4.65 per share next year. The company pledges to increase its dividend by 7 percent per year, indefinitely. If you require a return of 11 percent on your investment, how much will you pay for the company's stock today?

Using the constant growth model, we find the price of the stock today is: P0 = D1 / (R - g) P0 = $4.65 / (.11 - .07) P0 = $116.25

Suppose a stock had an initial price of $90 per share, paid a dividend of $2.10 per share during the year, and had an ending share price of $74.50. Compute the percentage total return What was the dividend yield? What was the capital gains yield?

Using the equation for total return, we find: R = [($74.50 - 90) + 2.10] / $90 R = -.1489, or -14.89% And the dividend yield and capital gains yield are: Dividend yield = $2.10 / $90 Dividend yield = .0233, or 2.33% Capital gains yield = ($74.50 - 90) / $90 Capital gains yield = -.1722, or -17.22%

Gesto, Inc., has an issue of preferred stock outstanding that pays a $4.50 dividend every year, in perpetuity If this issue currently sells for $79.85 per share, what is the required return? Explanation: The price of a share of preferred stock is the dividend divided by the required return. This is the same equation as the constant growth model, with a dividend growth rate of zero percent. Remember, most preferred stock pays a fixed dividend, so the growth rate is zero. This is a special case of the dividend growth model where the growth rate is zero, or the level perpetuity equation.

Using this equation, we find the price per share of the preferred stock is: R = D/P0 R = $4.50/$79.85 R = .0564 or 5.64%

Which ethical norm considers whether a particular act optimizes the benefits to those who are affected by it?

Utility

Suppose you just bought an annuity with 10 annual payments of $16,000 at the current interest rate of 12.5 percent per year. What happens to the value of your investment if interest rates suddenly drop to 7.5 percent?

Value of investment $ 109,825.30 ± 0.1%

Suppose you just bought an annuity with 10 annual payments of $16,000 at the current interest rate of 12.5 percent per year. What is the value of the investment at the current interest rate of 12.5 percent?

Value of investment $ 88,582.89 ± .1%

Suppose you just bought an annuity with 10 annual payments of $16,000 at the current interest rate of 12.5 percent per year. What happens to the value of your investment if interest rates suddenly rise to 17.5 percent?

Value of investment $73,202.07 ± 0.1%

Standard deviation is a measure of which one of the following?

Volatility

This is a written guarantee from the manufacturer that specifies the conditions under which the product can be returned, replaced, or repaired.

Warranty

A stock has an expected return of 16.1 percent, the risk-free rate is 6.45 percent, and the market risk premium is 7.2 percent. What must the beta of this stock be?

We are given the values for the CAPM except for the β of the stock. We need to substitute these values into the CAPM, and solve for the β of the stock. One important thing we need to realize is that we are given the market risk premium. The market risk premium is the expected return of the market minus the risk-free rate. We must be careful not to use this value as the expected return of the market. Using the CAPM, we find: E(Ri) = Rf + [E(RM) - Rf] × βi .161 = .0645 + .072βi βi = 1.340

Choose the correct statement regarding diversification.

We can eliminate most of our firm specific risk by holding a portfolio of 50 stocks from a variety of industries

Suppose you know that a company's stock currently sells for $65.20 per share and the required return on the stock is 10 percent. You also know that the total return on the stock is evenly divided between capital gains yield and dividend yield. If it's the company's policy to always maintain a constant growth rate in its dividends, what is the current dividend per share?

We know the stock has a required return of 10 percent, and the dividend and capital gains yield are equal, so: Dividend yield = 1/2(.10) Dividend yield = .050 = Capital gains yield Now we know both the dividend yield and capital gains yield. The dividend is simply the stock price times the dividend yield, so: D1 = .050($65.20) D1 = $3.26 This is the dividend next year. The question asks for the dividend this year. Using the relationship between the dividend this year and the dividend next year: D1 = D0(1 + g) We can solve for the dividend that was just paid: $3.26 = D0(1 + .050) D0 = $3.26 / 1.050 D0 = $3.10

Suppose you know that a company's stock currently sells for $65.50 per share and the required return on the stock is 12 percent. You also know that the total return on the stock is evenly divided between capital gains yield and dividend yield. If it's the company's policy to always maintain a constant growth rate in its dividends, what is the current dividend per share?

We know the stock has a required return of 12 percent, and the dividend and capital gains yield are equal, so: Dividend yield = 1/2(.12) Dividend yield = .060 = Capital gains yield Now we know both the dividend yield and capital gains yield. The dividend is simply the stock price times the dividend yield, so: D1 = .060($65.50) D1 = $3.93 This is the dividend next year. The question asks for the dividend this year. Using the relationship between the dividend this year and the dividend next year: D1 = D0(1 + g) We can solve for the dividend that was just paid: $3.93 = D0(1 + .060) D0 = $3.93 / 1.060 D0 = $3.71

Madeline Rollins is trying to decide whether she can afford a loan she needs in order to go to chiropractic school. Right now Madeline is living at home and works in a shoe store, earning a gross income of $980 per month. Her employer deducts a total of $180 for taxes from her monthly pay. Madeline also pays $120 on several credit card debts each month. The loan she needs for chiropractic school will cost an additional $180 per month. Calculate her debt payments-to-income ratio with and without the college loan. (Remember the 20 percent rule.)

With college loan: Debt payments-to-income ratio = Total debt payments / Net take-home pay = ($120 + $180) / ($980 - $180) = 0.3750, or 37.50% Without college loan: Debt payments-to-income ratio = Total debt payments / Net take-home pay = $120 / ($980 - $180) = 0.1500, or 15.00% According to the 20 percent rule, she cannot afford the college loan at this time. However, if she can pay off her credit cards, then she can afford to go to school.

Bonds that have been issued by state or local governments

What are municipal bonds?

-OTC dealers are connected electronically. -The OTC has no designated physical location.

What are some features of the OTC market for bonds?

Pay $800 today and receive $1,000 at the end of 5 years

What are the cash flows involved in the purchase of a 5-year zero-coupon bond that has a par value of $1,000 if the current price is $800? Assume the market rate of interest is 5 percent.

The real rate of return Expected future inflation The interest rate risk premium

What are the three components that influence the Treasury yield curve?

-private issue -public issue

What are the two major forms of long-term debt?

Sometimes short-term rates are higher and sometimes long-term rates are higher.

What does historical data suggest about the nature of short-term and long-term interest rates?

Par value Yield to Maturity Coupon rate Time remaining maturity

What four variables are required to calculate the value of a bond?

Current yield = annual coupon payment / current price

What is a bond's current yield?

-YTM is the expected return for an investor who buys the bond today and holds it to maturity. -YTM is the prevailing market interest rate for bonds with similar features.

What is a corporate bond's yield to maturity (YTM)?

are bonds that sell for less than the face value

What is a discount bond?

-it is a rate of return that has been adjusted for inflation -it is a percentage change in buying power

What is a real rate of return?

R = r + h

What is the equation for approximating the nominal rate of return? R = the nominal rate of interest r = the real rate of interest h = the inflation rate

It is the additional return demanded by investors to compensate for expected inflation

What is the inflation premium?

It is the actual percentage change in the dollar value of an investment unadjusted for inflation.

What is the nominal rate of return on an investment?

-a put bond -a convertible bond -a CoCo bond

Which of the following are bonds that have actually been issued?

it is known as the par value and is the principal amount repaid at maturity

Which of the following are true about a bond's face value?

They are issued by both corporations and governments They are interest-only loans

Which of the following are true of bonds?

No coupon payments are made on the bonds

Which of the following is true of zero coupon bonds?

-time to maturity -par value -coupon rate

Which of the following terms apply to a bond?

-Inflation premium -Real interest rate -Interest rate risk premium

Which three components determine the shape of the term structure of interest rates?

Louise McIntyre's monthly gross income is $3,500. Her employer withholds $750 in federal, state, and local income taxes and $300 in Social Security taxes per month. Louise contributes $150 each month for her IRA. Her monthly credit payments for VISA and MasterCard are $90 and $85, respectively. Her monthly payment on an automobile loan is $350. A. What is Louise's debt payments-to-income ratio? b. Is Louise living within her means?

a. Net income = $3,500 - $750 - $300 - $150 = $2,300 Monthly debt payments = VISA + MasterCard + Car loan = $90 + $85 + $350 = $525 Debt payments-to-income ratio = Total debt payments / After-tax income = $525 / $2,300 = 0.2283, or 22.83% b. Experts suggest that you spend no more than 20% of your net (after-tax) income on consumer credit payments.

When considering the federal income tax treatment for housing, the following is tax deductible:

interest on mortgage loans

When considering the federal income tax treatment for housing, the following...

interest on mortgage loans

indexed bond

interest rate paid is based upon the rate of inflation

IFRS

international financial reporting standards: a principle based set of international accounting standards stating how particular types of transactions and other events should be reported.

The relationship between the present value and the investment time period is best described as:

inverse.

When we estimate the best-case, worst-case, and base-case cash flows and calculate the corresponding NPVs, we are engaging in:

scenario analysis; asking what-if questions

Private placement

securities are offered and sold directly to limited number of investors

When using _______________, all of the variables except one are frozen in order to determine how sensitive the NPV estimate is to changes in that particular variable.

sensitivity analysis

The depreciation tax ____ is the tax savings that results from the depreciation deduction.

shield

Which one of the following categories of securities had the highest average return for the period 1926-2010?

small company stocks

Which one of the following categories of securities has had the most volatile returns over the period 1926-2010?

small-company stocks

Which does the term "in-line tenants" refer to?

smaller stores in a mall that are not anchor tenants

Which of the following is true relative to capital rationing?

soft rationing is typically internal in that the firm allocates funds to divisions for capital projects hard rationing implies the firm is unable to raise funds for projects

Which of the following types of venue is favored by small-business retailers?

specialty shop

The principle of diversification tells us that:

spreading an investment across many diverse assets will eliminate some of the total risk.

According to the _________ principle, once the incremental cash flows from a project have been identified, the project can be viewed as a "minifirm."

stand-alone

two types of investment risk

stand-alone risk, portfolio risk

adding stocks

standard deviation portfolio decreases as s stocks are added because they would not be perfectly correlated with the existing portfolio. eventually the diversification benefits of adding stocks dissipates (after 40 stocks) and for large stock portfolios . tends to converge to 20%

Equity

stockholder's investment in the firm and the cumulative profits retained in the business up to the date of the balance sheet


Set pelajaran terkait

The exclusivist approach to the question of religious diversity is the only approach that takes seriously the importance of religious truth.'—Explain and critically assess this claim.

View Set

Models of Addiction: Finals Questions

View Set

Intro to Weather and Climate (Exam 3)

View Set

Mastering Biology 3 - Mendelian Genetics

View Set

CGS power point chapters 1-3 TEST 2

View Set

Lecture 7 - Fundamentals of Machine Learning & Intro to Keras

View Set

Sociology Exam over Lessons 5 & 7.

View Set

Chapters 1 and 2 Introduction to Psychology and Research Methods

View Set